You are on page 1of 59

Exam Section : Item 1 of 50 National Board of Medical Examiners

■ Mark Medicine Self-Assessment

X 1. A 42-year-old man has had the gradual onset of greasy, diffuse scaling of the scalp with variable itching over the past year. Yellowish red scaling papules appear along the hairline and behind the ears; there is no hair loss. Which
of the following is the most likely diagnosis?

A) Exfoliative dermatitis
B) Lichen simplex chronicus
C) Pediculosis capitis
D) Psoriasis
E) Seborrheic dermatitis
Correct Answer: E.

Seborrheic dermatitis is a chronic inflammatory skin disease, which occurs in regions containing sebaceous glands. In adults, it typically presents as yellowish-red, scaly patches of the scalp and forehead, around the
ears, nasolabial folds, and chest. In infants, it presents as "cradle cap," with the appearance of a diaper rash and yellow scaling on the scalp. Diffuse, severe seborrheic dermatitis can occur in immunosuppressed
patients. Treatment consists of zinc pyrithione or selenium sulfide shampoo applied to the scalp along with topical antifungals such as ketoconazole. Topical corticosteroids can also be used for severe cases to
decrease the inflammatory reaction and pruritus.

Incorrect Answers: A, B, C, and D.

Exfoliative dermatitis (Choice A) is a severe inflammatory skin condition characterized by the development of generalized, diffuse erythema, and scaling with desquamation involving large areas of the body. This often
occurs in the setting of pre-existing skin disorders (eg, psoriasis, eczema), cancer (eg, leukemia, lymphoma), or after administration of certain medications (eg, sulfa, penicillin, phenytoin).

Lichen simplex chronicus (Choice B) is a localized region of skin thickening (lichenification) that results from repetitive rubbing or itching of the skin.

Pediculosis capitis (Choice C) refers to infestation of the scalp and hair by the Pedicu/us humanus capitis (human louse), which can cause severe pruritus. Treatment includes permethrin, pyrethrins, ivermectin, or
malathion lotions.

Psoriasis (Choice D) results in erythematous plaques with silvery scales that are typically located on extensor surfaces (e.g., knees, elbows). It can involve the scalp, but this is less common.

" , ~ F' r ,
Next Score Report
https://t.me/USMLENBME2CK Lab Values Calculator Help Pause
Exam Section : Item 2 of 50 National Board of Medical Examiners
■ Mark Medicine Self-Assessment

X 2. A 24-year-old man comes to the physician for a follow-up examination . Two months ago, he was seen for a red , scaly rash over his groin. A 6-week course of topical clotrimazole resolved the rash completely, but it recurred 2 days
ago. He is sexually active with one partner and uses condoms consistently. Examination shows an erythematous, excoriated rash over the groin and a similar rash over the instep of the left foot. A KOH preparation of a skin
scraping shows hyphae. Which of the following is the most likely cause of this patient's recurrent infection?

A) Autoinfection
B) Clotrimazole resistance
C) Impaired cellular immunity
D) Impaired humeral immunity
E) Reinfection from a sexual partner
Correct Answer: A.

Autoinfection of the patient's groin resulting in recurrent tinea cruris is the most likely scenario. Tinea (dermatophytosis) refers to cutaneous fungal infection with organisms such as Microsporum, Trichophyton, and
Epidermophyton. These are characterized by branching septated hyphae on KOH preparation of skin scrapings. The patient exhibits a red, scaly rash in his groin (tinea cruris), which is subsequently treated with first-
line therapy of clotrimazole. The patient also has a similar rash on his left foot (tinea pedis), which was not treated and likely served as a source for autoinfection to other regions of the body, resulting in the patient's
recurrent tinea cruris. Treatment with clotrimazole to both the groin and left foot at the same time is needed to appropriately treat the patient's multifocal tinea.

Incorrect Answers: B, C, D, and E.

Clotrimazole resistance (Choice B), while possible, is not common in dermatophytosis. Additionally, the patient's tinea cruris was appropriately treated with the clotrimazole, but the region was reinfected after
autoinfection from the patient's untreated tinea pedis.

Impaired cellular immunity (Choice C) and impaired humeral immunity (Choice D) refer to an underlying immunocompromised state. While immunocompromised patients are at increased risk for disseminated
dermatophytosis infections, the more likely scenario for this patient is autoinfection from an additional untreated region.

Reinfection from a sexual partner (Choice E) is possible, although less likely given the patient's untreated tinea pedis source. Dermatophytes spread through contact with infected skin, including from other individuals.

r " , ~ r-- r ,
Previous Next Score Report
https://t.me/USMLENBME2CK Lab Values Calculator Help Pause
Exam Section : Item 3 of 50 National Board of Medical Examiners
■ Mark Medicine Self-Assessment

X 3. An otherwise healthy 30-year-old man has had recurrent aching right shoulder pain initiated by reaching overhead; it also occurs at night in bed . The pain is most prominent in the area of the deltoid muscle. It is elicited by
abduction of the shoulder against resistance . Which of the following is the most likely site of the underlying condition?

A) Biceps tendon
B) Cervical nerve root
C) Deltoid muscle
D) Proximal humerus
E) Su w aspinatus tendon
Correct Answer: E.

The supraspinatus tendon is the most common location of rotator cuff injury. The supraspinatus muscle lies within the supraspinous fossa along the superior portion of the scapula, and its tendon inserts on the greater
tuberosity of the humerus. The supraspinatus muscle contributes to initiation of arm abduction prior to the action of the deltoid muscle. Injury to the supraspinatus tendon may occur from degeneration, impingement,
or trauma, and can be evaluated through abduction of the shoulder against resistance.

Incorrect Answers: A, B, C, and D.

The biceps tendon (Choice A) consists of a short head originating from the coracoid process and a long head originating from the intra-articular supraglenoid tubercle. The biceps brachii inserts at the radial tuberosity.
Injury to the biceps tendon results in weakness of elbow flexion and forearm supination.

Cervical nerve root (Choice B) injury would result in neurologic symptoms such as weakness, numbness, and paresthesia along a specific dermatome and myotome. It would not be aggravated by shoulder abduction.

The deltoid muscle (Choice C) is not commonly injured, much less than the supraspinatus tendon, and only contributes to abduction of the shoulder after the first 15 degrees.

Proximal humerus (Choice D) injury such as a surgical neck fracture can lead to axillary nerve injury, which would result in dysfunction of the deltoid muscle and loss of sensation over the proximal lateral arm.

r " , ~ r-- r ,
Previous Next Score Report
https://t.me/USMLENBME2CK Lab Values Calculator Help Pause
Exam Section : Item 4 of 50 National Board of Medical Examiners
■ Mark Medicine Self-Assessment

X 4. A 72-year-old woman comes to the physician because she is concerned about her high blood pressure. Two days ago, her blood pressure was 200/105 mm Hg at the drug store. She has a 20-year history of hypertension that
had been well controlled with a thiazide diuretic. At her last visit 3 months ago, her blood pressure was 140/75 mm Hg. Today, her pulse is 80/min, respirations are 12/min, and blood pressure is 210/114 mm Hg. Funduscopic
examination shows arteriovenous nicking. A right carotid bruit is heard. Laboratory studies are within normal limits. Which of the following is the most likely cause of these findings?

A) Acute glomerulonephritis
B) Acute porphyria
C) Coarctation of the aorta
D) Cushing syndrome
E) Hyperaldosteronism
F) Hyperparathyroidism
G) Hyperthyroidism
H) Pheochromocytoma
Renal artery stenosis
Correct Answer: I.

Renal artery stenosis is a cause of secondary hypertension related to abnormal stimulation of the juxtaglomerular apparatus secondary to stenosis of the renal artery leading to insufficient afferent blood flow. This
results in excessive production of renin and angiotensin. Secondary hypertension should be considered in new-onset or treatment-resistant hypertension. There are two main causes of renal artery stenosis:
fibromuscular dysplasia and atherosclerosis. In younger patients, fibromuscular dysplasia is the most common cause, while older patients develop renal artery stenosis more commonly from atherosclerosis. This
patient exhibits a carotid bruit, likely also secondary to atherosclerosis. Diagnosis is established with renal artery Doppler ultrasonography or magnetic resonance angiography. Treatment involves angioplasty or
stenting of the stenosed renal artery to improve flow. ACE inhibitors can be considered for unilateral stenosis but can lead to acute end-stage kidney disease in the setting of bilateral renal artery stenosis.

Incorrect Answers: A, B, C, D, E, F, G, and H.

Acute glomerulonephritis (Choice A) refers to a variety of glomerular diseases, including nephritic and nephrotic syndromes. Nephritic syndrome can result in hypertension, although this is less likely given the
patient's risk factors for atherosclerosis.

Acute porphyria (Choice B) presents with abdominal pain, polyneuropathy, psychological symptoms, and port wine-colored urine, not resistant hypertension.

Coarctation of the aorta (Choice C) is a cause of secondary hypertension related to narrowing of the aorta, although it most commonly presents in childhood with differential blood pressure between upper and lower
extremities. It is associated with bicuspid aortic valve and gonadal dysgenesis 45,X (Turner syndrome).

Cushing syndrome (Choice D) results from increased cortisol levels and presents with hypertension, moon facies, abdominal striae, osteoporosis, hyperglycemia, and amenorrhea. This patient demonstrates risk
factors for atherosclerosis, with no additional findings to suggest Cushing syndrome.

Hyperaldosteronism (Choice E) presents with hypertension and metabolic derangements, including hypokalemia and metabolic alkalosis. This can occur in the setting of an adrenal adenoma such as in Conn
syndrome or from bilateral adrenal hyperplasia.

Primary hyperparathyroidism (Choice F) is a potential cause of secondary hypertension and may also present with hypercalcemia and hypophosphatemia, with symptoms such as constipation, renal stones,
psychiatric disturbances, and osseous lytic lesions.

Hyperthyroidism (Choice G) can result in secondary hypertension but also typically presents with heat intolerance, weight loss, diarrhea, hyperreflexia, palpitations, and myopathy.
r " , ~ r-- r ,
Previous Next Score Report
https://t.me/USMLENBME2CK Lab Values Calculator Help Pause
Exam Section : Item 4 of 50 National Board of Medical Examiners
■ Mark Medicine Self-Assessment

examination shows arteriovenous nicking. A right carotid bruit is heard. Laboratory studies are within normal limits. Which of the following is the most likely cause of these findings?

A) Acute glomerulonephritis
B) Acute porphyria
C) Coarctation of the aorta
D) Cushing syndrome
E) Hyperaldosteronism
F) Hyperparathyroidism
G) Hyperthyroidism
H) Pheochromocytoma
Renal artery stenosis
Correct Answer: I.

Renal artery stenosis is a cause of secondary hypertension related to abnormal stimulation of the juxtaglomerular apparatus secondary to stenosis of the renal artery leading to insufficient afferent blood flow. This
results in excessive production of renin and angiotensin. Secondary hypertension should be considered in new-onset or treatment-resistant hypertension. There are two main causes of renal artery stenosis:
fibromuscular dysplasia and atherosclerosis. In younger patients, fibromuscular dysplasia is the most common cause, while older patients develop renal artery stenosis more commonly from atherosclerosis. This
patient exhibits a carotid bruit, likely also secondary to atherosclerosis. Diagnosis is established with renal artery Doppler ultrasonography or magnetic resonance angiography. Treatment involves angioplasty or
stenting of the stenosed renal artery to improve flow. ACE inhibitors can be considered for unilateral stenosis but can lead to acute end-stage kidney disease in the setting of bilateral renal artery stenosis.

Incorrect Answers: A, B, C, D, E, F, G, and H.

Acute glomerulonephritis (Choice A) refers to a variety of glomerular diseases, including nephritic and nephrotic syndromes. Nephritic syndrome can result in hypertension, although this is less likely given the
patient's risk factors for atherosclerosis.

Acute porphyria (Choice B) presents with abdominal pain, polyneuropathy, psychological symptoms, and port wine-colored urine, not resistant hypertension.

Coarctation of the aorta (Choice C) is a cause of secondary hypertension related to narrowing of the aorta, although it most commonly presents in childhood with differential blood pressure between upper and lower
extremities. It is associated with bicuspid aortic valve and gonadal dysgenesis 45,X (Turner syndrome).

Cushing syndrome (Choice D) results from increased cortisol levels and presents with hypertension, moon facies, abdominal striae, osteoporosis, hyperglycemia, and amenorrhea. This patient demonstrates risk
factors for atherosclerosis, with no additional findings to suggest Cushing syndrome.

Hyperaldosteronism (Choice E) presents with hypertension and metabolic derangements, including hypokalemia and metabolic alkalosis. This can occur in the setting of an adrenal adenoma such as in Conn
syndrome or from bilateral adrenal hyperplasia.

Primary hyperparathyroidism (Choice F) is a potential cause of secondary hypertension and may also present with hypercalcemia and hypophosphatemia, with symptoms such as constipation, renal stones,
psychiatric disturbances, and osseous lytic lesions.

Hyperthyroidism (Choice G) can result in secondary hypertension but also typically presents with heat intolerance, weight loss, diarrhea, hyperreflexia, palpitations, and myopathy.

Pheochromocytoma (Choice H) results in episodic hypertension with associated palpitations, diaphoresis, headache, and pallor.

r " , ~ r-- r ,
Previous Next Score Report
https://t.me/USMLENBME2CK Lab Values Calculator Help Pause
Exam Section : Item 5 of 50 National Board of Medical Examiners
■ Mark Medicine Self-Assessment

X 5. A 57-year-old woman with stage IV non-small cell lung carcinoma is admitted to the hospital because of a 2-day history of shortness of breath and fever. Her temperature is 39°C (102.2°F). She receives the diagnosis of
postobstructive pneumonia. Treatment with intravenous antibiotics is begun, and her symptoms resolve 3 days later. During a family meeting to discuss her treatment options, the patient states that she wishes to discontinue
chemotherapy and requests no resuscitation if she has a cardiac arrest. In addition to obtaining a signed advance directive and durable power of attorney for health care decisions, which of the following is the most appropriate
recommendation for this patient?

A) Discharge home with home hosl)ice care


B) Discharge home with physical and occupational therapy
C) Discharge home with visiting nurse services
D) Discharge to a skilled nursing care facility
E) Remain in the hospital
Correct Answer: A.

Discharge home with home hospice care is indicated for patients whose life expectancy is less than six months in the setting of a terminal diagnosis. This patient presents with stage IV non-small cell lung carcinoma
and wishes to discontinue aggressive treatment such as chemotherapy. Hospice care focuses on palliation of a patient's symptoms with attention to providing comfort, rather than aggressive treatment in an attempt to
cure the patient's underlying illness. Given the patient's current prognosis and stated wishes, hospice care would be the most appropriate recommendation.

Incorrect Answers: B, C, D, and E.

Discharge home with physical and occupational therapy (Choice B) is indicated for the rehabilitation of patients following hospitalization, surgery, and/or trauma in order to improve the patient's physical abilities in an
attempt to return to their daily life and work activities. This patient demonstrates the wish to discontinue aggressive therapy for her cancer, making hospice a better arrangement.

Discharge home with visiting nurse services (Choice C) and discharge to a skilled nursing care facility (Choice D) are indicated for patients discharged from the hospital who need some level of nursing care to assist
with the handling of their medical conditions. These services provide nursing care to assist with medications and hygiene but would not focus primarily on palliation and patient comfort like hospice care.

Remain in the hospital (Choice E) would not be the best choice for this patient as it would not allow the patient to receive palliative care in the comfort of her own home.

r " , ~ r-- r ,
Previous Next Score Report
https://t.me/USMLENBME2CK Lab Values Calculator Help Pause
Exam Section : Item 6 of 50 National Board of Medical Examiners
■ Mark Medicine Self-Assessment

X 6. An 87-year-old resident of a skilled nursing care facility is brought to the physician by staff because of a lesion on his left leg for 3 weeks . He has been confined to bed for 3 years because of a left hemiplegia following a cerebral
infarction. Examination shows a 5 x 7-cm, grade 3 pressure ulcer over the left greater trochanter; there is an overlying eschar with a small rim of erythema. There are no other areas of ulceration. Which of the following is the most
appropriate next step in management?

A) Use of an air bed


B) Application of wet-to-dry dressings
C) Application of silver sulfadiazine cream and a sterile dressing
D) Debridement of the ulce
E) Skin graft
CorrectAnswer: D.

Debridement of the ulcer is the next best step in the management of a decubitus ulcer with findings of necrotic tissue. The patient presents with a grade 3 pressure ulcer, which is characterized by full-thickness tissue
loss with visible subcutaneous fat. Additionally, this patient demonstrates an overlying eschar and rim of erythema. To allow for appropriate healing of the wound, the necrotic tissue should be debrided. This should be
followed by the application of wet-to-dry dressing, pressure offloading, and consistent wound cleaning.

Incorrect Answers: A, B, C, and E.

Use of an air bed (Choice A) can be utilized to limit pressure to dependent body regions, especially in patients with decreased mobility. This would serve as a preventive measure for the development of pressure
ulcers, but only as adjunct management of an existing ulcer with evidence of necrosis.

Application of wet-to-dry dressings (Choice B) and application of silver sulfadiazine cream and a sterile dressing (Choice C) are therapeutic interventions used for the treatment of decubitus ulcers to promote tissue
regrowth and maintain cleanliness of the wound. These are not sufficient interventions in isolation for a decubitus ulcer with evidence of necrosis.

Skin grafts (Choice E) are treatment choices for patients with clean stage 3 or 4 ulcers that do not respond to optimal medical interventions or when the patient's quality of life would improve from rapid wound closure.

r " , ~ r-- r ,
Previous Next Score Report
https://t.me/USMLENBME2CK Lab Values Calculator Help Pause
Exam Section : Item 7 of 50 National Board of Medical Examiners
■ Mark Medicine Self-Assessment

X 7. A 42-year-old woman comes to the physician for a follow-up examination . She was diagnosed with hypertension 6 months ago, and hydrochlorothiazide therapy was begun. Eight weeks ago, her blood pressure was
176/96 mm Hg, and lisinopril was added to her regimen. She says she feels well. She has no other history of serious illness. She occasionally takes ibuprofen for muscle pain or headache. She walks for 30 minutes three times
weekly. She is 170 cm (5 ft 7 in) tall and weighs 66 kg (145 lb); BMI is 23 kg/m 2• Her pulse is 76/min , and blood pressure is 160/98 mm Hg in the right upper extremity and 162/96 mm Hg in the left upper extremity while sitting
and standing . Cardiopulmonary examination shows no abnormalities. There is no peripheral edema. Fasting laboratory studies show:
Serum
Na+ 140 mEq/L
K+ 4 mEq/L
Cl· 105 mEq/L
HC0 3- 24 mEq/L
Urea nitrogen 16 mg/dl
Glucose 94 mg/dl
Creatinine 1 mg/dl
Urine protein 1+

The patient asks how she can decrease her risk for renal failure. Which of the following is the most appropriate recommendation?

A) Protein-restricted diet
B) 24-Hour urine collection for measurement of protein concentration
C Increasing the dose of lisino ril
D) Switching from hydrochlorothiazide to furosemide
E) No further measures are indicated at this time
Correct Answer: C.

Increasing the dose of lisinopril would be the most appropriate recommendation to decrease the patient's risk for renal failure (end-stage kidney disease). Lisinopril is an ACE inhibitor, which inhibits angiotensin
converting enzyme resulting in decreased production of angiotensin II and aldosterone. This results in reduced blood pressure by multiple mechanisms. Additionally, the decreased production of angiotensin II results
in relative efferent arteriole dilatation. This leads to decreased glomerular filtration pressure. The patient demonstrates chronic hypertension, which is a leading cause of end-stage kidney disease. Increasing the
dose of lisinopril will assist in further decreasing the patient's systemic blood pressure in addition to efferent arteriole dilatation, both of which will reduce intraglomerular pressure. This reduction in intraglomerular
pressure will reduce damage to the glomerular capillaries and decrease her risk for end-stage kidney disease.

lncorrectAnswers:A, B, D, and E.

Protein-restricted diet (Choice A) has been shown to reduce progression of nephropathy in patients with chronic end-stage kidney disease, especially in diabetic nephropathy. This patient's risk for end-stage kidney
disease is mostly related to her chronic hypertension for which increasing the dose of lisinopril would demonstrate the most benefit.

24-Hour urine collection for measurement of protein concentration (Choice B) is utilized for the evaluation of nephrotic syndrome and albuminuria for diabetic nephropathy. The patient's nephropathy is likely from
chronic hypertension and her urinalysis demonstrates 1+ urine protein, which is a low-level of urine protein. Measurement of 24-hour urine protein would not further assist in her management.

Switching from hydrochlorothiazide to furosemide (Choice D) would not appreciably decrease the patient's risk for end-stage kidney disease as these agents do not demonstrate reno-protective properties and are
both diuretics. Furosemide is typically used in the management of hypervolemia, while hydrochlorothiazide is a first-line treatment for hypertension management.

No further measures are indicated at this time (Choice E) is not the correct answer choice, as the patient's blood pressure remains increased and she would benefit from an increased lisinopril dose to improve her
blood pressure and reduce her risk for end-stage kidney disease.

r " , ~ r-- r ,
Previous Next Score Report
https://t.me/USMLENBME2CK Lab Values Calculator Help Pause
Exam Section : Item 8 of 50 National Board of Medical Examiners
■ Mark Medicine Self-Assessment

X 8. A 55-year-old man is admitted to the hospital in a coma following a sudden collapse. He has a history of recent coronary bypass operation for coronary artery disease. ECG shows wide QRS complexes (QRS 160 ms) with a
ventricular rate of 170/min and an atrial rate of 110/min. Which of the following is the most appropriate initial step in management?

A) Administration of digitalis
B) Administration of verapamil
C) Carotid sinus massage
D) Direct current countershock
E) External pacing
CorrectAnswer: D.

Direct current countershock refers to the delivery of electrical energy for the treatment of cardiac arrhythmias. There are two forms of direct current countershock: synchronized cardioversion and non-synchronized
defibrillation. The patient presents with a wide-QRS tachyarrhythmia and altered mental status. The differential for wide-QRS tachyarrhythmias includes ventricular tachycardia (monomorphic or polymorphic),
antidromic atrioventricular reentrant tachycardia in Wolff-Parkinson-White syndrome, or supraventricular tachycardia with aberrant conduction. Because of the patient's unstable tachyarrhythmia, the most appropriate
initial treatment is the administration of a direct current countershock. Indications for direct current countershock include tachyarrhythmias presenting with no pulse, hypotension, end-organ failure, or altered mental
status.

Incorrect Answers: A, B, C, and E.

Administration of digitalis (Choice A) is indicated for the treatment of atrial fibrillation through inhibiting conduction through the atrioventricular (AV) node or for the treatment of heart failure by increasing cardiac
myocyte contractility through inhibition of the sodium-potassium ATPase.

Administration of verapamil (Choice B) is primarily used for treatment of supraventricular tachyarrhythmias such as atrial fibrillation or flutter.

Carotid sinus massage (Choice C) increases vagal output resulting in an increased AV node refractory period and can be utilized as a first-line attempt to slow supraventricular tachyarrhythmias such as AV nodal
reentrant tachycardia. It does not have a common role in the treatment of atrial fibrillation or flutter, sinus tachycardia, or wide-complex tachycardia.

External pacing (Choice E) refers to the administration of transcutaneous electrical energy for the treatment of refractory or unstable bradycardia.

r " , ~ r-- r ,
Previous Next Score Report
https://t.me/USMLENBME2CK Lab Values Calculator Help Pause
Exam Section : Item 9 of 50 National Board of Medical Examiners
■ Mark Medicine Self-Assessment

9. A 32-year-old woman is admitted to the hospital 8 hours after the onset of nausea and vomiting and epigastric abdominal pain that radiates to her back. She has
had two similar episodes during the past 2 months. She has no other history of serious illness. She does not drink alcohol. Abdominal examination shows
moderate epigastric tenderness . Laboratory studies show:
Hemoglobin 12.5 g/dL
Hematocrit 38%
Leukocyte count 15,400/mm3
Platelet count 326,000/mmJ
Serum amylase 1450 U/L

Ultrasonography of the gallbladder shows no abnormalities. A CT scan of the abdomen is shown . Which of the following is the most appropriate next step in
diagnosis?

A) HIDAscan
B Endoscopic retrograde cholangiopancreatograP.hY
C) Esophagogastroduodenoscopy
D) Mesenteric angiography
E) Percutaneous transhepatic cholangiography
Correct Answer: B.

Endoscopic retrograde cholangiopancreatography (ERCP) is the next most appropriate step for this patient given its diagnostic and therapeutic
function. The patient presents following recurrent episodes of nausea, vomiting, and epigastric pain, potentially secondary to recurrent episodes of
acute pancreatitis. The CT scan of the abdomen demonstrates a pseudocyst in the pancreatic parenchyma. Pancreatic pseudocysts are a potential
complication of acute pancreatitis and are granulation tissue lined collections of simple fluid located in the pancreatic parenchyma or in the
peripancreatic region. Pancreatic pseudocysts can result in persistent elevation of serum pancreatic enzymes and recurrent bouts of epigastric
abdominal pain. Intervention is indicated for pseudocysts if they are larger than 5 cm or are symptomatic. The first-line intervention for drainage of the
pseudocysts is endoscopic by either a transpapillary approach or through creation of a cystogastrostomy or cystoduodenostomy. These result in the
creation of drainage pathways between the cyst and gastrointestinal tract.

lncorrectAnswers:A, C, D, and E.

HIDA scan (Choice A) is used for the diagnosis of acute cholecystitis. If there is lack of visualization of the gallbladder after administration of the
radioactive tracer, this indicates the presence of acute cholecystitis with blockage of the cystic duct secondary to an impacted, non-mobile gallstone.

Esophagogastroduodenoscopy (Choice C) alone would not allow for definitive diagnosis and therapeutic intervention of the pseudocyst. Symptomatic
pseudocysts larger than 5 cm in diameter require drainage through either a transpapillary approach or cystogastrostomy/cystoduodenostomy.

Mesenteric angiography (Choice D) is indicated for definitive evaluation of the mesenteric vessels. This may be performed in the setting of mesenteric
ischemia with the goal to localize and treat an obstruction to improve mesenteric blood flow or in the setting of a gastrointestinal bleed, which can be
treated with vascular embolization by interventional radiology.

Percutaneous transhepatic cholangiography (Choice E) is indicated for visualization of the biliary tree to further delineate findings following ERCP or
magnetic resonance cholangiopancreatography (MRCP), or if prior ERCP was unsuccessful in cannulation of the duodenal papilla. Additionally, this
technique can be used for therapeutic placement of biliary drains or stents.

r " , ~ r-- r ,
Previous Next Score Report
https://t.me/USMLENBME2CK Lab Values Calculator Help Pause
Exam Section : Item 1o of 50 National Board of Medical Examiners
■ Mark Medicine Self-Assessment

X 10. A 27-year-old man comes to the physician because of fatigue and loose stools over the past 3 months; during this period , he has had a 6.8-kg (15-lb) weight loss despite a good appetite and normal dietary intake. He has a
childhood history of diarrhea and difficulty gaining weight that resolved without treatment during early adolescence. He has not traveled recently. He does not appear acutely ill. He is 173 cm (5 ft 8 in) tall and weighs 57 kg
(125 lb); BMI is 19 kg/m 2• Vital signs are within normal limits. Examination shows no abnormalities. Test of the stool shows no leukocytes, and cultures of the stool are negative for pathogens. A 72-hour test of the stool for fecal
fat shows 12 g/d (N=2-6). D-xylose absorption test shows a serum D-xylose concentration of 4 mg/dl (N=25-40) 2 hours after the administration of 25 g of D-xylose. Which of the following is the most likely underlying cause of
this patient's malabsorption?

A) Bacterial overgrowth in the small bowel


B) Exocrine pancreatic insufficiency
C) Failure of bile acid excretion
D) Failure of bile acid reabsorption
E) Lactase deficiency
F) Villous atrophy in the small bowel
Correct Answer: F.

Villous atrophy in the small bowel is a characteristic finding of celiac disease. Celiac disease is a gluten-sensitive enteropathy which is mediated through immune intolerance of the protein gliadin, commonly found in
wheat products. This results in inflammatory infiltration of the proximal small bowel, which leads to characteristic intraepithelial lymphocytosis, villous atrophy, and crypt hyperplasia. Malabsorption of substances in the
proximal small bowel ensues, which presents symptomatically as chronic diarrhea, steatorrhea, and nutritional deficiencies. Abnormal small bowel absorption results in abnormal serum D-xylose and fecal fat tests. D-
xylose is absorbed passively in the proximal small bowel; reduced serum D-xylose indicates reduced small bowel mucosal surface area for absorption. The increased concentration of fecal fat indicates intestinal
malabsorption of lipids, also seen in the presence of abnormal small bowel mucosa.

Incorrect Answers: A, B, C, D, and E.

Bacterial overgrowth in the small bowel (Choice A) occurs in the setting of impaired intestinal peristalsis, which allows for the uncontrolled growth of bacteria caused by the loss of appropriate peristaltic propulsion.
This can result in intestinal malabsorption, diarrhea, bloating, and chronic abdominal pain. The diagnosis of bacterial overgrowth occurs through the hydrogen breath test.

Exocrine pancreatic insufficiency (Choice B) occurs in the setting of chronic pancreatitis, and results in insufficient pancreatic enzyme production and secretion. This leads to inadequate digestion of macromolecules
and resultant malnutrition, presenting with diarrhea and steatorrhea.

Failure of bile acid excretion (Choice C) and reabsorption (Choice D) would result in the development of steatorrhea secondary to impaired fat digestion and absorption. Cholestasis and structural causes of biliary
obstruction can result in impaired bile acid excretion. Bile acids are absorbed in the terminal ileum. Absorption can be impaired by pathologies affecting the terminal ileum, such as Crohn disease.

Lactase deficiency (Choice E) results in dietary intolerance to lactose, a carbohydrate found in human and cow milk. Patients present with abdominal cramping, bloating, and diarrhea following ingestion of dairy
products.

r " , ~ r-- r ,
Previous Next Score Report
https://t.me/USMLENBME2CK Lab Values Calculator Help Pause
Exam Section : Item 11 of 50 National Board of Medical Examiners
■ Mark Medicine Self-Assessment

X 11 . A 77-year-old woman comes to the physician because of palpitations for 2 days. She has had a 4.5-kg (10-lb) weight loss during the past 6 months. Her temperature is 37.4°C (99.3°F), pulse is 120/min and irregular, respirations
are 24/min , and blood pressure is 130/70 mm Hg. Examination shows nontender enlargement of the left lobe of the thyroid gland. Serum studies show a thyroid-stimulating hormone concentration less than 0.05 µU/ml and
thyroxine {T 4) concentration of 30 µg/dl. A 131 1study shows a single area of increased uptake in the left lobe of the thyroid gland and suppressed uptake elsewhere in the gland . Which of the following is the most likely
mechanism of these findings?

A) Autoimmune destruction of thyroid cells


B) Focal granulomatous inflammation
C) Parafollicular cell cytokine overproduction
D) Production of thyroid-stimulating immunoglobulin
E) Autonomous T 4 !)reduction
Correct Answer: E.

Autonomous T 4 production is the most likely mechanism for the patient's hyperthyroidism. The patient presents with weight loss, palpitations, and tachycardia, which are typical symptoms of hyperthyroidism. Lab
values demonstrate a low thyroid-stimulating hormone {TSH) concentration and increased thyroxine {T 4) concentration. This indicates the patient has primary hyperthyroidism, with the origin of hyperthyroidism likely
within her thyroid gland. On physical exam, there is a nontender left thyroid lobe enlargement with a single "hot" nodule on radioactive iodine uptake {RAIU) scan. These findings are consistent with a toxic adenoma,
which autonomously produces thyroxine resulting in clinical hyperthyroidism.

Incorrect Answers: A, B, C, and D.

Autoimmune destruction of thyroid cells {Choice A) occurs in the setting of chronic lymphocytic (Hashimoto) thyroiditis, an autoimmune disorder that presents with anti-thyroid peroxidase and anti-thyroglobulin
antibodies. In the acute phase, chronic lymphocytic (Hashimoto) thyroiditis can present with hyperthyroidism because of underlying follicular cell damage and release of thyroid hormone, although this would present
with diffusely decreased 1-131 uptake on an RAIU scan.

Focal granulomatous inflammation {Choice B) refers to subacute {de Quervain) thyroiditis, which is a self-limited thyroiditis that typically presents with tender thyroid enlargement and is associated with flu-like
symptoms.

Parafollicular cell cytokine overproduction {Choice C) refers to the production of calcitonin by the parafollicular cells, which can be abnormally increased in the setting of medullary thyroid cancer. This would typically
not affect thyroid hormone production except in the case of advanced medullary thyroid cancer infiltrating adjacent functional thyroid tissue. This scenario could present with hypothyroidism from insufficient thyroid
hormone production.

Production of thyroid-stimulating immunoglobulin {Choice D) occurs in Graves disease which results in primary hyperthyroidism with associated exophthalmos and pretibial myxedema. RAIU typically demonstrates
diffuse 1-131 uptake without a focal nodule.

r " , ~ r-- r ,
Previous Next Score Report
https://t.me/USMLENBME2CK Lab Values Calculator Help Pause
Exam Section : Item 12 of 50 National Board of Medical Examiners
■ Mark Medicine Self-Assessment

X 12. A 72-year-old nursing home resident is brought to the emergency department because of abdominal pain and bloody stools for 3 days. She has dementia, Alzheimer type , and a long-standing history of constipation treated with
psyllium and docusate. She is in no apparent distress. Her temperature is 39°C (102.2°F), pulse is 88/min , respirations are 18/min, and blood pressure is 150/80 mm Hg. Abdominal examination shows left lower quadrant
tenderness and distention. Rectal examination shows blood and no masses. She is not oriented to place or time . Which of the following is the most likely diagnosis?

A) Campylobacteriosis G) Irritable bowel syndrome


B) Celiac disease H) lschemic colitis
C) C/ostridium difficile colitis I) Salmonellosis
D) Colon cancer J) Shigellosis
E) Diverticulitis K) Ulcerative colitis
F) Diverticulosis

Correct Answer: E.

Diverticulitis is a common cause of left lower quadrant abdominal pain and tenderness in adults. It often presents with fever, nausea, and vomiting. It results from inflammation and infection involving pre-existing diverticula.
Hematochezia can be present secondary to concomitant diverticular bleeding. Complications of diverticulitis include perforation leading to peritonitis, abscess formation, bowel obstruction, and fistula formation. Treatment includes
bowel rest, symptomatic control, and antibiotics covering enteric flora.

Incorrect Answers: A, B, C, D, F, G, H, I, J, and K.

Campylobacteriosis (Choice A), Salmonellosis (Choice 1), and Shigellosis (Choice J) are all causes of bloody diarrhea following certain exposures. This patient presents with focal left lower quadrant pain without diarrhea, making
acute bacterial infection less likely.

Celiac disease (Choice B) is a chronic inflammatory condition mostly affecting the proximal small bowel in response to gluten ingestion. This results in chronic diarrhea, bloating, weight loss, and malnutrition.

C/ostridium difficile colitis (Choice C) is an opportunistic infection that most commonly occurs after the use of antibiotics because of the effect on the patient's colonic microbiota. It typically presents with abdominal pain and severe
watery diarrhea, with the risk for complications such as fulminant colitis and toxic megacolon.

Colon cancer (Choice D) is a cause of hematochezia, although it would not be expected to present acutely with fever.

Diverticulosis (Choice F) indicates the presence of colonic diverticula, which do not produce abdominal pain outside of episodes of diverticulitis, but can result in diverticular bleeding, which is one of the most common causes of
painless hematochezia.

Irritable bowel syndrome (Choice G) presents with intermittent abdominal pain that is relieved by bowel movements and associated alterations in bowel habits with intermittent episodes of diarrhea and/or constipation.

lschemic colitis (Choice H) refers to inadequate blood supply to the colon, which can lead to bowel wall necrosis, loss of immune barriers, and bowel perforation. This most commonly occurs in the setting of atherosclerosis, or
hypercoagulable states with emboli such as atrial fibrillation, resulting in diminished blood flow within a vascular territory or a watershed region when it occurs following systemic hypotension.

Ulcerative colitis (Choice K) is a type of inflammatory bowel disease that results in mucosal and submucosal ulceration and inflammation that progresses retrograde along the colon from the rectum. This typically presents with
chronic bloody diarrhea, tenesmus, and abdominal pain.

r " , ~ r-- r ,
Previous Next Score Report
https://t.me/USMLENBME2CK Lab Values Calculator Help Pause
Exam Section : Item 13 of 50 National Board of Medical Examiners
■ Mark Medicine Self-Assessment

X 13. A 27-year-old man is brought to the emergency department 40 minutes after the sudden onset of shortness of breath while scuba diving. He reports that he had to ascend rapidly from a 70-foot depth because he was running out
of compressed air. During the ascent, he had the sudden onset of midchest pain ; he had shortness of breath by the time he reached the surface. He has no history of serious illness and takes no medications. He is in moderate
respiratory distress and unable to speak in complete sentences. His respirations are 28/min. Pulse oximetry on room air shows an oxygen saturation of 88%. He is using accessory muscles of respiration . The trachea is midline.
Pulmonary examination shows decreased breath sounds and hyperresonance to percussion on the left. The remainder of the examination shows no abnormalities. Which of the following is the most appropriate next step in
diagnosis?

A) Measurement of serum D-dimer concentration


B) Chest x-ray
C) ECG
D) Echocardiography
E) CT scan of the thorax
Correct Answer: B.

Chest x-ray is indicated for the evaluation of nontension pneumothorax. The patient presents after rapid ascent during scuba diving, which places him at increased risk for decompression sickness, which can have
multiple manifestations. Decompression sickness occurs when dissolved gases precipitate out of solution in the body and form bubbles or when gas expands because of the rapid decrease in ambient pressure.
These mechanisms increase the risk for air emboli, arthropathy, tympanic membrane rupture, and pneumothoraces. On physical exam of this patient, there are decreased breath sounds and hyperresonance to
percussion in the left hemithorax, which indicates the presence of a pneumothorax. The evaluation for a pneumothorax includes a chest x-ray. Depending on the size of the pneumothorax, the patient may require
thoracostomy for appropriate management.

lncorrectAnswers:A, C, D, and E.

Measurement of serum o-dimer concentration (Choice A) is recommended for evaluation of patients with low pretest probability of deep venous thrombosis (DVT) or pulmonary em bolus. If the o-dimer is increased, the
next appropriate step would be ultrasound and Doppler evaluation of the lower extremity veins for a DVT or CT chest angiography for evaluation of a pulmonary embolism.

ECG (Choice C) is indicated for evaluation of cardiac arrhythmias and acute coronary syndromes. The patient presents with midchest pain, which is likely secondary to his left-sided pneumothorax, not myocardial
ischemia/infarction. The patient does not exhibit any risk factors for an acute coronary syndrome.

Echocardiography (Choice D) entails ultrasound evaluation of the heart, allowing for visualization of pericardia! pathology and cardiac morphology, motion, ejection fraction, and valvular function. Lung ultrasound, not
cardiac, is useful in the evaluation of pneumothorax.

CT scan of the thorax (Choice E) can be performed to evaluate for a variety of thoracic pathology, both acute and chronic. This patient demonstrates physical exam findings concerning for a nontension left-sided
pneumothorax, which would be more rapidly evaluated using a chest x-ray rather than CT scan of the chest.

r " , ~ r-- r ,
Previous Next Score Report
https://t.me/USMLENBME2CK Lab Values Calculator Help Pause
Exam Section : Item 14 of 50 National Board of Medical Examiners
■ Mark Medicine Self-Assessment

X 14. A 62-year-old man is brought to the emergency department 45 minutes after the sudden onset of shortness of breath. When paramedics arrived at his home, he was agitated but alert and oriented to person , place, and time . He
was cyanotic and in severe respiratory distress. He was treated with 100% oxygen by face mask en route to the hospital. He has a 15-year history of chronic obstructive pulmonary disease treated with home oxygen therapy
(1 .5 L/min) over the past 2 years. He has smoked two packs of cigarettes daily for 40 years. On arrival , he is drowsy and awakens only to loud voices or painful stimuli. His temperature is 37°C (98.6°F), pulse is 104/min,
respirations are 14/min and shallow, and blood pressure is 164/90 mm Hg . Breath sounds are diffusely decreased , and air movement is poor with prolonged expiration . Which of the following is the most likely underlying cause of
this patient's decreased level of consciousness?

A) Hypercarbia
B) Hypoxemia
C) Metabolic acidosis
D) Oxygen toxicity
E) Respiratory alkalosis
Correct Answer: A.

Hypercarbia is the most likely underlying cause of the patient's decreased level of consciousness. The patient has a history of chronic obstructive pulmonary disease (COPD). COPD is an obstructive lung disease
characterized by decreased lung function from a combination of features of chronic bronchitis and/or emphysema, resulting in airflow obstruction on expiration. The patient is presenting with a COPD exacerbation,
characterized by shortness of breath and shallow respirations. His presentation is likely further exacerbated by the administration of 100% oxygen by face mask, which in a patient with COPD can theoretically result in
depression of respiratory drive and further worsening of the ventilation/perfusion mismatch. Depressed ventilation can result in acute hypercarbia, which can lead to an acute decrease in level of consciousness.

Incorrect Answers: B, C, D, and E.

Hypoxemia (Choice B) does not likely account for the patient's decreased level of consciousness, as he was treated with 100% oxygen by face mask. This can result in elevation of the patient's oxygen saturation
above his normal baseline from underlying COPD, which can theoretically lead to reduced respiratory drive and worsening of the ventilation/perfusion mismatch.

Metabolic acidosis (Choice C) can occur in the setting of multiple causes, either with or without an associated anion gap. This patient presents with an acute COPD exacerbation with depressed ventilation, whereas
metabolic acidosis is typically followed by increased ventilation via respiratory compensation.

Oxygen toxicity (Choice D) can occur after the supplemental administration of high concentration oxygen, which results in overproduction of reactive oxygen species, and can manifest with primarily neurotoxic
symptoms such as vision and hearing changes, dizziness, nausea, altered mental status, and convulsions.

Respiratory alkalosis (Choice E) occurs in the setting of hyperventilation. The patient presents with a COPD exacerbation and inadequate ventilation, which results in hypercarbia and respiratory acidosis.

r " , ~ r-- r ,
Previous Next Score Report
https://t.me/USMLENBME2CK Lab Values Calculator Help Pause
Exam Section : Item 15 of 50 National Board of Medical Examiners
■ Mark Medicine Self-Assessment

X 15. A 47-year-old woman with type 2 diabetes mellitus comes to the physician for a follow-up examination . Six weeks ago, her fasting serum glucose concentration was 177 mg/dl; urine protein was 1+. She has been taking
metformin and following a low-carbohydrate diet for the past 4 weeks. She has no other medical problems and is taking no other medications. Her father died of end-stage renal disease secondary to type 2 diabetes mellitus.
She is 165 cm (5 ft 5 in) tall and weighs 82 kg (180 lb); BMI is 30 kg/m 2• Her weight is unchanged from her last visit. Today, her blood pressure is 130/82 mm Hg; at her last examination 4 weeks ago, it was 128/80 mm Hg. The
remainder of the physical examination shows no abnormalities. Laboratory studies show:
Hemoglobin A 1c 7.3%
Serum
Urea nitrogen 17 mg/dl
Glucose 169 mg/dl
Creatinine 1.1 mg/dl
Urine protein 180 mg/24 h

Which of the following measures is most likely to slow the progression of diabetic nephropathy in this patient?

A) Calorie-reduced diet
B) Dietary protein restriction to 0.6 g/kg daily
C) Maintenance of a hemoglobin A 1c of less than 6%
D) Addition of lisinopril to the medication regimen
E) Switching from metformin to insulin therapy
CorrectAnswer: D.

Addition of lisinopril to the medication regimen is the most likely measure to slow the progression of the patient's diabetic nephropathy. Diabetic nephropathy occurs following nonenzymatic glycosylation of the
glomerular basement membrane and efferent arterioles. This results in an abnormally increased glomerular filtration rate that can cause progressive hyperfiltration injury to the glomerulus over time. This initially
presents as microalbuminuria, which can subsequently progress to macroalbuminuria, and then end-stage kidney disease. The addition of an ACE inhibitor such as lisinopril to the patient's regimen results in
decreased angiotensin II production, which causes relative efferent arteriole dilatation. This decreases constriction of the efferent arteriole secondary to nonenzymatic glycosylation, reducing hyperfiltration injury of the
glomerulus and slowing progression of the patient's diabetic nephropathy.

Incorrect Answers: A, B, C, and E.

Calorie-reduced diet (Choice A) does not directly affect the progression of diabetic nephropathy. While it may result in weight loss and improved glucose control in this patient with diabetes, the addition of lisinopril will
more directly slow progression of diabetic nephropathy.

Dietary protein restriction to 0.6 g/kg daily (Choice B) does demonstrate evidence for slowing the rate of diabetic nephropathy progression, although this is not as effective as ACE inhibitor therapy.

Maintenance of a hemoglobin A 1c of less than 6% (Choice C) and switching from metformin to insulin therapy (Choice E) would improve management of the patient's type 2 diabetes mellitus, which can slow the
progression of microvascular complications such as neuropathy, retinopathy, and nephropathy. While this would slow the progression of diabetic nephropathy, the more effective method for slowing progression is to
add an ACE inhibitor to the patient's medication regimen.

r " , ~ r-- r ,
Previous Next Score Report
https://t.me/USMLENBME2CK Lab Values Calculator Help Pause
Exam Section : Item 16 of 50 National Board of Medical Examiners
■ Mark Medicine Self-Assessment

X 16. A 20-year-old woman comes for a routine health maintenance examination. Her temperature is 37°C (98.6°F), pulse is 80/min, respirations are 12/min, and blood pressure is 110/70 mm Hg. The S 2 is split and fixed . A grade 2/6
systolic murmur is heard at the third intercostal space. Examination shows no other abnormalities. Which of the following is the most likely diagnosis?

A) Aortic insufficiency
B) Atrial septal defect
C) Mitral insufficiency
D) Patent ductus arteriosus
E) Pulmonic insufficiency
F) Tetralogy of Fallo!
G) Tricuspid insufficiency
H) Ventricular septal defect
Correct Answer: B.

Atrial septal defect is a defect in the interatrial septum. The most common type is an ostium secundum defect, although ostium primum defects are commonly associated with trisomy 21. The atrial septal defect results
in a left-to-right shunt with abnormal flow of blood from the left atrium to the right atrium, resulting in relative volume overload of the right atrium and ventricle. This increased stroke volume of the right ventricle results
in delayed closure of the pulmonic valve, which presents as a fixed, split S 2, and low-grade physiologic ejection murmur on cardiac auscultation. If the atrial septal defect remains uncorrected, it can result in the
development of Eisenmenger syndrome secondary to prolonged pulmonary vasculature remodeling resulting in pulmonary arterial hypertension and shunt reversal leading to cyanosis.

Incorrect Answers: A, C, D, E, F, G, and H.

Aortic insufficiency (Choice A) presents with an early diastolic decrescendo murmur best heard in the right second intercostal space and is most commonly associated with endocarditis, rheumatic fever, and aortic root
dilation.

Mitral insufficiency (Choice C) presents with a holosystolic murmur best heard in the left fourth or fifth intercostal space along the midclavicular line and radiates to the left axilla. It is commonly associated with mitral
valve prolapse and prior myocardial infarction.

Patent ductus arteriosus (Choice D) results in a continuous, machine-like murmur best heard in the left second intercostal space and radiates to the clavicle.

Pulmonic insufficiency (Choice E) presents with a diastolic murmur best heard in the left second intercostal space.

Tetralogy of Fallo! (Choice F) consists of pulmonary infundibular stenosis, overriding aorta, ventricular septal defect, and right ventricular hypertrophy, resulting in a right-to-left shunt with cyanosis first noted in infancy.
The murmur heard with Tetralogy of Fallo! relates to the underlying ventricular septal defect (Choice H), characterized by a holosystolic murmur best heard in the lower left sternal border.

Tricuspid insufficiency (Choice G) demonstrates a holosystolic murmur best heard in the lower left sternal border.

r " , ~ r-- r ,
Previous Next Score Report
https://t.me/USMLENBME2CK Lab Values Calculator Help Pause
Exam Section : Item 17 of 50 National Board of Medical Examiners
■ Mark Medicine Self-Assessment

X 17. A 52-year-old woman with hypertension comes to the emergency department 45 minutes after the onset of tearing pain that starts at the base of her neck and radiates to her back. On arrival, her temperature is 37°C (98.6°F),
pulse is 96/min, respirations are 20/min, and blood pressure is 190/110 mm Hg. Pulse oximetry on room air shows an oxygen saturation of 99% . No other abnormalities are noted on physical examination . An ECG shows sinus
tachycardia and left ventricular hypertrophy. A chest x-ray shows a widened mediastinum . The most appropriate next step in management is administration of which of the following?

A) Labetalol
B) Morphine
C) Nitroglycerin
D) Phenylephrine
E) Sodium nitroprusside
Correct Answer: A.

Labetalol is a first-line antihypertensive medication used to lower blood pressure in the setting of presumed aortic dissection. The patient presents with lower neck pain radiating to her back, hypertensive emergency,
and a widened mediastinum on a chest x-ray. These are all findings associated with acute aortic dissection. A CT angiography of the thoracic aorta is indicated to further delineate whether the patient exhibits a type A
or B aortic dissection, as this will dictate appropriate management. Regardless of which type of aortic dissection the patient has, it is imperative to appropriately treat the patient's hypertensive emergency.
Hypertensive emergency is defined as a blood pressure greater than 180/120 with evidence of end-organ damage (eg, aortic dissection). Intravenous antihypertensive medications should be used for the treatment of
hypertensive emergency to appropriately reduce blood pressure and limit progression of end-organ damage. In aortic dissection, to prevent the advancement of the dissection flap and resultant occlusion of branch
vessels, the blood pressure should be rapidly reduced using a beta-blocker to lower cardiac output and diminish fluctuations in pulse pressure, combined with an alpha blocker or arterial vasodilator to lower mean
arterial pressure by reducing vascular resistance. Labetalol is a nonspecific adrenoreceptor blocker that targets both a 1- and 13 1-adrenoreceptors, making it an ideal choice.

Incorrect Answers: B, C, D, and E.

Morphine {Choice B) is indicated for the treatment of acute pain. While this patient demonstrates lower neck pain radiating to the back, it is more important to treat the underlying cause of the pain, which is
hypertensive emergency with acute aortic dissection.

Nitroglycerin {Choice C) can be used to help alleviate myocardial-induced chest pain by reducing cardiac preload and myocardial oxygen demand, but this medication would only be appropriate for use in aortic
dissection if accompanied by a beta-blocker to prevent reflex tachycardia.

Phenylephrine {Choice D) is a vasopressor that primarily functions through stimulation of a 1-adrenoreceptors, resulting in increased vasoconstriction, systemic vascular resistance, and systemic blood pressure. The
patient is presenting with hypertensive emergency and presumed aortic dissection, which require reduction of the patient's blood pressure.

Sodium nitroprusside {Choice E) is a vasodilator that can be utilized for the treatment of hypertensive emergency, although the better choice is labetalol because of its effect of lowering both vascular resistance and
heart rate, both of which are necessary to reduce worsening of the patient's presumed aortic dissection.

r " , ~ r-- r ,
Previous Next Score Report
https://t.me/USMLENBME2CK Lab Values Calculator Help Pause
Exam Section : Item 18 of 50 National Board of Medical Examiners
■ Mark Medicine Self-Assessment

X 18. A 62-year-old man has had lethargy, weakness, confusion, and constipation for 3 days.Three weeks ago, he was diagnosed with inoperable squamous cell carcinoma of the lung . Examination shows decreased skin turgor.
Breath sounds are decreased at the left apex. Laboratory studies show:
Leukocyte count 8100/mm3
Serum
Na+ 142 mEq/L
K+ 4.8 mEq/L
Ca 2+ 15.2 mg/dL
Phosphorus 1.2 mg/dL
Albumin 4.5 g/dL

An ECG shows a short QT interval. Intravenous administration of 0.9% saline(200 ml/h) is begun . Which of the following is the most appropriate pharmacotherapy?

A) Allopurinol F) Fresh frozen plasma


B) Cisplatin G) Heparin
C) Cryoprecipitate H) Hydroxyurea
D) Demeclocycline I) Pamidronate
E) Doxycycline J) Tamoxifen
Correct Answer: I.

Pamidronate is a bisphosphonate and is indicated for the treatment of hypercalcemia through osteoclast inhibition. Acute hypercalcemia commonly presents with constipation, lethargy, weakness, and altered mental status. It can
result in a short QT interval and nephrogenic diabetes insipidus, leading to significant dehydration. Common causes of hypercalcemia include hyperparathyroidism and various malignancies, especially squamous cell carcinoma of
the lung which can produce parathyroid hormone-related peptide (PTHrP). PTHrP induces osteoclast activity resulting in hypercalcemia. First-line therapy for hypercalcemia includes isotonic fluid resuscitation (0.9% saline)
followed by IV bisphosphonate and/or calcitonin administration if further calcium reduction is required.

Incorrect Answers: A, B, C, D, E, F, G, H, and J.

Allopurinol (Choice A) inhibits xanthine oxidase, decreasing production of uric acid, and is commonly used as preventative therapy for gout.

Cisplatin (Choice B) is a platinum chemotherapeutic agent that cross-links DNA and is used in the treatment of various malignancies.

Cryoprecipitate (Choice C) contains fibrinogen, Factors VIII and XIII, and von Willebrand factor. It is commonly used for treatment of coagulopathic states with diminished levels of fibrinogen and factor VIII.

Demeclocycline (Choice D) is a member of the tetracycline family that serves as an anti-diuretic hormone antagonist used in the treatment of refractory SIADH.

Doxycycline (Choice E) is also a member of the tetracycline family and is a bacteriostatic antibiotic used for a variety of bacterial infections.

Fresh frozen plasma (Choice F) contains numerous coagulation factors and is used for the treatment of urgent anticoagulation reversal (eg, warfarin), and hypocoagulable states (eg, disseminated intravascular coagulation and
cirrhosis).

Heparin (Choice G) binds anti-thrombin Ill and inhibits the activity of thrombin and factor Xa. It is commonly used for immediate anticoagulation in the setting of deep venous thrombosis, pulmonary embolism, and acute coronary
syndrome.

r " ,
. . ~ r-- r ,
Previous Next Score Report
https://t.me/USMLENBME2CK Lab Values Calculator Help Pause
Exam Section : Item 18 of 50 National Board of Medical Examiners
■ Mark Medicine Self-Assessment

Serum
Na+ 142 mEq/L
K+ 4.8 mEq/L
Ca 2+ 15.2 mg/dl
Phosphorus 1.2 mg/dl
Albumin 4.5 g/dl

An ECG shows a short QT interval. Intravenous administration of 0.9% saline(200 ml/h) is begun . Which of the following is the most appropriate pharmacotherapy?

A) Allopurinol F) Fresh frozen plasma


B) Cisplatin G) Heparin
C) Cryoprecipitate H) Hydroxyurea
D) Demeclocycline I) Pamidronate
E) Doxycycline J) Tamoxifen
Correct Answer: I.

Pamidronate is a bisphosphonate and is indicated for the treatment of hypercalcemia through osteoclast inhibition. Acute hypercalcemia commonly presents with constipation, lethargy, weakness, and altered mental status. It can
result in a short QT interval and nephrogenic diabetes insipidus, leading to significant dehydration. Common causes of hypercalcemia include hyperparathyroidism and various malignancies, especially squamous cell carcinoma of
the lung which can produce parathyroid hormone-related peptide (PTHrP). PTHrP induces osteoclast activity resulting in hypercalcemia. First-line therapy for hypercalcemia includes isotonic fluid resuscitation (0.9% saline)
followed by IV bisphosphonate and/or calcitonin administration if further calcium reduction is required.

Incorrect Answers: A, B, C, D, E, F, G, H, and J.

Allopurinol (Choice A) inhibits xanthine oxidase, decreasing production of uric acid, and is commonly used as preventative therapy for gout.

Cisplatin (Choice B) is a platinum chemotherapeutic agent that cross-links DNA and is used in the treatment of various malignancies.

Cryoprecipitate (Choice C) contains fibrinogen, Factors VIII and XIII, and von Willebrand factor. It is commonly used for treatment of coagulopathic states with diminished levels of fibrinogen and factor VIII.

Demeclocycline (Choice D) is a member of the tetracycline family that serves as an anti-diuretic hormone antagonist used in the treatment of refractory SIADH.

Doxycycline (Choice E) is also a member of the tetracycline family and is a bacteriostatic antibiotic used for a variety of bacterial infections.

Fresh frozen plasma (Choice F) contains numerous coagulation factors and is used for the treatment of urgent anticoagulation reversal (eg, warfarin), and hypocoagulable states (eg, disseminated intravascular coagulation and
cirrhosis).

Heparin (Choice G) binds anti-thrombin Ill and inhibits the activity of thrombin and factor Xa. It is commonly used for immediate anticoagulation in the setting of deep venous thrombosis, pulmonary embolism, and acute coronary
syndrome.

Hydroxyurea (Choice H) is used in the treatment of sickle cell disease because of its induction of increased fetal hemoglobin production.

Tamoxifen (Choice J) is a selective estrogen receptor modulator used for the treatment of breast cancer.

r " , ~ r-- r ,
Previous Next Score Report
https://t.me/USMLENBME2CK Lab Values Calculator Help Pause
Exam Section : Item 19 of 50 National Board of Medical Examiners
■ Mark Medicine Self-Assessment

X 19. A 52-year-old man comes to the physician because of fatigue for 4 months. He has hypertension and a 2-year history of chronic renal insufficiency. His serum creatinine concentrations have ranged between 2.5 mg/dl and
3.5 mg/dl. Medications include lisinopril and amlodipine. His pulse is 96/min, and blood pressure is 160/72 mm Hg. Examination shows no other abnormalities except for mild pallor. Test of the stool for occult blood is negative.
Laboratory studies show:
Hematocrit 22%
Mean corpuscular volume 89 µm 3
Reticulocyte count 2.5%
Serum
Creatinine 3.4 mg/dl
Ferritin 150 ng/ml
Iron 35 µg/dl
Transferrin saturation 28% (N=22%-46%)

Supplementation with which of the following is most likely to have prevented these hematologic findings?

A) E[Ylhroi:1oieti
B) Felic acid
C) Iron
D) Vitamin B 12 (cyanocobalamin)
E) No preventive measures are available
Correct Answer: A.

Erythropoietin is a hormone produced by the interstitial cells in the renal peritubular capillary bed. It is upregulated in response to hypoxia and stimulates proliferation of erythrocytic progenitors in the bone marrow. In
the setting of chronic renal disease, there is insufficient production of erythropoietin by the kidneys, resulting in diminished blood marrow stimulation and subsequent normocytic, nonhemolytic anemia. This is
characterized by a normal mean corpuscular volume with diminished reticulocyte count and normal ferritin and transferrin saturation. Supplemental administration of erythropoietin to patients with chronic renal
insufficiency can prevent and treat the development of normocytic anemia.

Incorrect Answers: B, C, D, and E.

Felic acid (Choice B) supplementation can be used for the treatment of megaloblastic anemia secondary to folic acid deficiency and as preventive supplementation for patients at increased risk for folic acid deficiency,
such as individuals with hemolytic anemia (eg, sickle cell disease), alcohol use disorder, and pregnancy.

Iron (Choice C) supplementation is indicated for the treatment of iron deficiency anemia, which is characterized by microcytic mean corpuscular volume with reduced transferrin saturation and low ferritin.

Vitamin B 12 (cyanocobalamin) (Choice D) supplementation is utilized for the treatment of megaloblastic anemia secondary to cobalamin deficiency. It can also be taken as a preventive supplement for individuals with
risk factors such as veganism, pernicious anemia, Crohn disease, or history of previous gastrectomy or gastric bypass.

No preventive measures (Choice E) is not the correct answer, as supplementation with erythropoietin would prevent the development of anemia in this patient with chronic end-stage kidney disease.

r " , ~ r-- r ,
Previous Next Score Report
https://t.me/USMLENBME2CK Lab Values Calculator Help Pause
Exam Section : Item 20 of 50 National Board of Medical Examiners
■ Mark Medicine Self-Assessment

20. The life table shown compares the natural disease history of groups A and B. Which group at which time has the poorest 1-year survival rate? 100
A) Group A at Time 0-1
80
B) Group Bat Time 0-1 .;
> 60
C) Group A at Year 2-3 ·~
::,
fl) 40
D) GrouP. B at Year 3-4 ~ B
E) Group B at Year 4-5 20
CorrectAnswer: D. 0
0 2 3 4 5
Group B at Year 3-4 demonstrates the largest decrease in percent survival for a 1-year time span. The life table plots the natural disease progression for the Years
percent of individuals alive over time (years) for Group A and Group B. At Year 3, Group B demonstrates a percent survival of approximately 80%. By Year 4,
this decreases to approximately 50%, indicating a 1-year drop in percent survival of 30%.

Incorrect Answers: A, B, C, and E.

Group A at Time 0-1 and Year 2-3 (Choices A and C) demonstrate a linear decrease in percent survival of approximately 10% per year.

Group Bat Time 0-1 (Choice B) demonstrates no appreciable decrease in percent survival.

Group Bat Year 4-5 (Choice E) demonstrates a percent survival decrease of approximately 5%, less than Group B at Year 3-4.

r " , ~ r-- r ,
Previous Next Score Report
https://t.me/USMLENBME2CK Lab Values Calculator Help Pause
Exam Section : Item 21 of 50 National Board of Medical Examiners
■ Mark Medicine Self-Assessment

X 21 . A 67-year-old man had the acute onset of bilateral weakness of the lower extremities and urinary incontinence 2 days ago. He has had carcinoma of the prostate for 2 years and bone metastases to the pelvis for 6 months. His
temperature is 37.4 °C (99.3°F), pulse is 86/min, respirations are 18/min, and blood pressure is 160/86 mm Hg. There is tenderness to palpation over the lumbosacral spine most prominent at S1 . He is unable to lift his legs
against gravity; deep tendon reflexes are absent at the knees and ankles. Which of the following is the most appropriate initial step in diagnosis?

A) Radionuclide bone scan


B) Electromyography and nerve conduction studies
C) CT scan of the head
D) MRI of the lumbosacral spine
E) Lumbar puncture
CorrectAnswer: D.

MRI of the lumbosacral spine is indicated for evaluation of acute onset neurological deterioration from cauda equina syndrome. The patient has a history of osseous metastatic prostate cancer. Cauda equina
syndrome occurs from acute compression of the cauda equina nerve roots in the lumbosacral spinal canal, most commonly from an acute disc herniation, or a traumatic or pathologic vertebral column fracture. This
results in bilateral lower extremity weakness, loss of deep tendon reflexes, bowel or bladder dysfunction (eg, overflow urinary incontinence), and saddle anesthesia. Emergent MRI of the lumbosacral spine is indicated
to investigate the cause of cauda equina syndrome and to determine the necessity of surgical decompression.

Incorrect Answers: A, B, C, and E.

Radionuclide bone scan (Choice A) uses technetium 99m-methyl diphosphonate (Tc-99m MDP) to evaluate for the presence of osteoblastic osseous metastases, most commonly in the setting of breast and prostate
cancer. It can also be used for the evaluation of osteomyelitis, although this is less common.

Electromyography and nerve conduction studies (Choice B) are used to differentiate myopathic and neuropathic causes of extremity weakness, and would not be used in the emergent evaluation of acute weakness.

CT scan of the head (Choice C) is used to evaluate for a variety of intracranial pathologies, whereas this patient exhibits symptoms of cauda equina syndrome which likely originates from pathology in the lumbosacral
spine.

Lumbar puncture (Choice E) evaluates the composition of cerebrospinal fluid and is indicated for the evaluation of meningitis, encephalitis, subarachnoid hemorrhage, and other nervous system diseases (e.g.,
Guillain-Barre syndrome, idiopathic intracranial hypertension).

r " , ~ r-- r ,
Previous Next Score Report
https://t.me/USMLENBME2CK Lab Values Calculator Help Pause
Exam Section : Item 22 of 50 National Board of Medical Examiners
■ Mark Medicine Self-Assessment

X 22. A 72-year-old man has had burning , aching pain in the distal extremities for 3 weeks ; the pain is exacerbated by lowering the extremities and relieved by elevation. Examination shows tenderness and swelling of the fingers and
wrist, knee , ankle, and toe joints; the overlying skin is warm and erythematous. There is clubbing of the fingers and toes . Which of the following is most likely to be abnormal?

A) Serum creatinine concentration


B) Serum ferritin concentration
C) Serum protein electrophoresis
D) X-ray of the abdomen
E) X-ray of the chest
Correct Answer: E.

X-ray of the chest is indicated for evaluation of the presence of a primary lung carcinoma. The patient presents with para neoplastic hypertrophic osteoarthropathy, which is a syndrome of digit clubbing and long-bone
periostitis. It can manifest with symptoms of burning, aching pain with tenderness, and swelling of the distal extremities along with clubbing of the fingers and toes. The most common cause of paraneoplastic
hypertrophic osteoarthropathy is non-small cell lung cancer, specifically adenocarcinoma. X-ray of the chest should be performed to evaluate for a primary lung mass.

Incorrect Answers: A, B, C, and D.

Serum creatinine concentration (Choice A) would evaluate for the presence of acute or chronic end-stage kidney disease. Chronic end-stage kidney disease can result in renal osteodystrophy characterized by
subperiosteal bone thinning, not digit clubbing.

Serum ferritin concentration (Choice B) is used in the evaluation of anemias. Increased serum ferritin can be seen in hemochromatosis, which can present with atypical joint pain and calcium pyrophosphate deposition
disease.

Serum protein electrophoresis (Choice C) is used in the evaluation of hemoglobinopathies and monoclonal gammopathies. Multiple myeloma presents with lytic bone lesions secondary to abnormally increased
osteoclast activation.

X-ray of the abdomen (Choice D) is often utilized for the evaluation of acute abdominal pathology (eg, small bowel obstruction), and would serve no purpose in the workup for a primary malignancy.

r " , ~ r-- r ,
Previous Next Score Report
https://t.me/USMLENBME2CK Lab Values Calculator Help Pause
Exam Section : Item 23 of 50 National Board of Medical Examiners
■ Mark Medicine Self-Assessment

X 23. A 62-year-old woman comes to the physician because of heartburn for 4 months and difficulty swallowing solids for 1 month . She has daily episodes of epigastric burning radiating to her chest and neck, especially at night or
after ingestion of large meals. Treatment with ranitidine over the past 3 months has been ineffective. She has not had nausea, vomiting, change in bowel habits, or weight loss. She takes no other medications. She does not
smoke or drink alcohol. She appears well nourished. Her temperature is 36.9°C (98.4°F), pulse is 80/min, respirations are 16/min, and blood pressure is 136/86 mm Hg. Examination shows no scleral icterus, jaundice, or
lymphadenopathy. Test of the stool for occult blood is negative. Her hemoglobin concentration is 13 g/dl, serum AST activity is 18 U/L, and serum lipase activity is 224 U/L (N=14-280). Which of the following is the most
appropriate next step in management?

A) Add metoclopramide to the medication regimen


B) Add omeprazole to the medication regimen
C) Switch to famotidine therapy
0) Switch to omeprazole therapy
E) Upperendoscopy
F) No further testing or pharmacotherapy indicated
Correct Answer: E.

Upper endoscopy is the most appropriate next step in management for gastroesophageal reflux disease (GERO) resistant to medication therapy and/or with the development of solid food dysphagia. GERO typically
presents with burning epigastric and lower chest pain that radiates superiorly and is exacerbated by the ingestion of large meals. First-line therapy for GERO is an empiric trial of proton pump inhibitors or H2 receptor
blockers. If the patient demonstrates reflux that is nonresponsive to medication therapy, an upper endoscopy should be performed for definitive evaluation of the esophagus, especially if the patient develops
dysphagia. Oysphagia may indicate the development of reflux-associated sequelae such as erosive or eosinophilic esophagitis, Barrett esophagus, or esophageal carcinoma.

lncorrectAnswers:A, B, C, 0, and F.

Adding metoclopramide to the medication regimen (Choice A) would not be the best choice, as metoclopramide is a 02 receptor antagonist that increases gastric contractility and motility. It is used primarily for the
treatment of gastroparesis and as an antiemetic. It can be used for persistent GERO because of its effect on increasing lower esophageal sphincter tone, but its potentially severe adverse effects (ie, parkinsonism,
tardive dyskinesia) mean this should only be used for severe, resistant cases of GERO.

Adding omeprazole to the medication regimen (Choice B) and switching to famotidine (Choice C) or omeprazole (Choice 0) are not appropriate next steps in management since the patient did not have a response to
empiric treatment with the histamine-blocking medication ranitidine and has developed solid food dysphagia.

Additionally, no further testing or pharmacotherapy (Choice F) is not appropriate since her GERO did not improve with empiric therapy. Because of her medication-resistant GERO, her esophagus needs to be
definitively evaluated for reflux-associated sequelae or alternative culprit lesions.

r " , ~ r-- r ,
Previous Next Score Report
https://t.me/USMLENBME2CK Lab Values Calculator Help Pause
Exam Section : Item 24 of 50 National Board of Medical Examiners
■ Mark Medicine Self-Assessment

X 24. A 37-year-old woman comes for a routine follow-up examination. She has renal failure secondary to polycystic kidney disease and is receiving hemodialysis. Immunization history includes diphtheria-tetanus toxoid and 23-valent
pneumococcal vaccine 3 years ago and influenza virus vaccine 12 months ago. Examination shows no abnormalities. Serum hepatitis B surface antibody assay is positive. Which of the following is the most appropriate vaccine
to administer at this visit?

A) Diphtheria-tetanus toxoid
B) Hepatitis B
C) Influenza virus
D) Meningococcal
E) 23-Valent pneumococcal
Correct Answer: C.

Influenza virus vaccine is recommended for all individuals aged 6 months or older on an annual basis. The patient is at high risk for complications from influenza given her underlying renal failure (end-stage kidney
disease) from polycystic kidney disease. The patient last received her influenza vaccine 12 months ago, indicating the influenza virus vaccine is the most appropriate vaccine to administer at the current visit.

lncorrectAnswers:A, B, D, and E.

The diphtheria-tetanus toxoid booster (Choice A) is recommended every 10 years for adults who have previously received the Tdap vaccine. The patient has already received this vaccine.

Hepatitis B vaccine (Choice B) is typically administered to newborns or to adults who previously did not receive the vaccine. The patient exhibits serum hepatitis B surface antibodies, which implies successful prior
immunization or a resolved prior infection, indicating she does not require the hepatitis B vaccine.

Meningococcal conjugate vaccine (Choice D) is indicated for all children, with an additional serogroup B vaccine indicated for all children over 10 years of age at increased risk for meningococcal disease (eg,
dormitory, military living).

The 23-valent pneumococcal vaccine (PPSV23) (Choice E) is indicated for all adults over 65 years of age and in patients with high-risk conditions such as chronic heart, renal, and liver disease or chronic smokers.
The patient has already received this vaccine.

r " , ~ r-- r ,
Previous Next Score Report
https://t.me/USMLENBME2CK Lab Values Calculator Help Pause
Exam Section : Item 25 of 50 National Board of Medical Examiners
■ Mark Medicine Self-Assessment

X 25. A 67-year-old man is brought to the emergency department because of a 2-day history of episodes of vomiting blood . He has no history of similar episodes. He has esophageal adenocarcinoma, hypertension , type 2 diabetes
mellitus, osteoarthritis, and coronary artery disease. He has a prosthetic, mechanical aortic valve. He is receiving radiation therapy. He notes that the therapy makes him tired and he naps several hours daily. His other
medications are lisinopril , metformin , glipizide, simvastatin, warfarin , and aspirin. Which of the following is the strongest indication for anticoagulant therapy in this patient?

A) Adenocarcinoma
B) Coronary artery disease
C) Decreased activity
D) Prosthetic aortic valve
E) Radiation therapy
CorrectAnswer: D.

Prosthetic aortic valve is the patient's strongest indication for anticoagulant therapy. The patient has a mechanical aortic valve, which is at substantial risk for thrombus formation without anticoagulation therapy.
Thrombus formation at the site of the mechanical aortic valve would place the patient at risk for acute arterial thromboembolic events such as stroke, myocardial infarction, and acute peripheral arterial occlusions, all
of which would result in significant morbidity and risk of mortality. For patients with mechanical aortic valves, the target international normalized ratio (INR) with warfarin therapy is 2.5 to 3.5.

Incorrect Answers: A, B, C, and E.

Adenocarcinoma (Choice A) increases the patient's risk for venous thromboembolic disease, most commonly deep venous thrombosis and pulmonary emboli. Patients with underlying adenocarcinoma are not
recommended to start anticoagulation without evidence of thromboembolic disease.

Coronary artery disease (CAD) (Choice B) increases the risk for myocardial infarction. Patients with CAD are recommended to take aspirin, an irreversible antiplatelet medication, to reduce the risk for myocardial
infarction. Anticoagulation is not indicated for patients with CAD without acute coronary syndrome.

Decreased activity (Choice C), specifically immobilization, also increases risk for thromboembolism. Prophylactic anticoagulation is often utilized in immobilized hospital and institutionalized patients to reduce the risk
for thromboembolism through the use of subcutaneous heparin or enoxaparin. This patient is indicated to receive therapeutic anticoagulation for his mechanical aortic valve.

Radiation therapy (Choice E) is not an indication for anticoagulation therapy as radiation therapy does not directly increase the risk for thromboembolic disease. It is often used in the treatment of a variety of diseases,
most commonly cancer.

r " , ~ r-- r ,
Previous Next Score Report
https://t.me/USMLENBME2CK Lab Values Calculator Help Pause
Exam Section : Item 26 of 50 National Board of Medical Examiners
■ Mark Medicine Self-Assessment

X 26. A 22-year-old man comes to the physician because of swelling of the neck for 1 month and malaise, fever, fatigue, night sweats, and generalized pruritus for 2 weeks . Acetaminophen has provided only minimal relief of his
symptoms. His temperature is 38.3°C (100.9°F), pulse is 90/min , respirations are 16/min, and blood pressure is 100/67 mm Hg . There is a 3 x 4-cm left anterior cervical lymph node. Conjunctivae are not injected , and the
tympanic membranes and pharynx are not erythematous. The lungs are clear to auscultation. A grade 1/6 , nonradiating systolic murmur is heard best at the upper right sternal border. Bowel sounds are slightly hyperactive.
Laboratory studies show:
Hemoglobin 14 g/dL
Leukocyte count 6000/mm 3
Segmented neutrophils 60%
Bands 2%
Eosinophils 1%
Lymphocytes 33%
Monocytes 4%

Which of the following is the most likely diagnosis?

A) Acute lymphocytic leukemia H) Infectious mononucleosis


B) Cat-scratch disease I) Rubella
C) Chronic lymphocytic leukemia J) Sarcoidosis
D) Drug reaction K) Syphilis
E) Gastric carcinoma L) Systemic lupus erythematosus
F) HIV infection M) Tuberculosis
G Hodgkin disease

Correct Answer: G.

Hodgkin disease is a type of lymphoma, predominantly B-cell in origin, that occurs most commonly in patients ages 20-30 years and 55-65 years. Patients typically present with lymphadenopathy along with nonspecific symptoms
such as fever, night sweats, and weight loss (B symptoms), hepatosplenomegaly, and pruritus. Pel-Ebstein fevers refer to recurrent high fevers for 1 to 2 weeks followed by 1- to 2-week afebrile periods. A specific sign for Hodgkin
disease associated lymphadenopathy is alcohol-induced pain at the enlarged lymph node sites. The most common sites for lymphadenopathy are the neck and mediastinum. Definitive diagnosis requires excisional lymph node
biopsy demonstrating Reed-Sternberg cells. The most common subtype is nodular sclerosing, and the worst prognosis occurs in the setting of the lymphocyte-depleted subtype.

Incorrect Answers: A, B, C, D, E, F, H, I, J, K, L, and M.

Acute lymphocytic leukemia (ALL) {Choice A) most commonly occurs in pediatric patients and presents with B symptoms and hepatosplenomegaly. It is defined as greater than 20% lymphoblasts on peripheral blood smears and
bone marrow biopsies. Down syndrome patients demonstrate an increased risk for the development of ALL.

Cat-scratch disease {Choice B) is caused by transmission of Bartone/la hense/ae through scratches from domestic or feral cats, typically presenting with lymphadenitis in the lymphatic drainage pattern of the scratch location.

Chronic lymphocytic leukemia {Choice C) most commonly occurs in adults greater than 60 years of age. Patients are often asymptomatic and demonstrate slow progression, with characteristic "smudge" cells on peripheral blood
smears.

Drug reaction {Choice D) would not typically present with a 1-month duration of malaise, fevers, and night sweats. While it can present with pruritus and diffuse rash, this is typically following the administration of the causative
drug .

., ., 1e11., le • 11 .. t • • ■ 1 I•,. ., • .. 11 .. 11111 1 .. t .. I .. I l l , It• ■ • .. •t 11 .... •

r " , ~ r-- r ,
Previous Next Score Report
https://t.me/USMLENBME2CK Lab Values Calculator Help Pause
Exam Section : Item 26 of 50 National Board of Medical Examiners
■ Mark Medicine Self-Assessment

B) Cat-scratch disease I) Rubella


C) Chronic lymphocytic leukemia J) Sarcoidosis
D) Drug reaction K) Syphilis
E) Gastric carcinoma L) Systemic lupus erythematosus
F) HIV infection M) Tuberculosis
G Hodgkin disease

Correct Answer: G.

Hodgkin disease is a type of lymphoma, predominantly B-cell in origin, that occurs most commonly in patients ages 20-30 years and 55-65 years. Patients typically present with lymphadenopathy along with nonspecific symptoms
such as fever, night sweats, and weight loss (B symptoms), hepatosplenomegaly, and pruritus. Pel-Ebstein fevers refer to recurrent high fevers for 1 to 2 weeks followed by 1- to 2-week afebrile periods. A specific sign for Hodgkin
disease associated lymphadenopathy is alcohol-induced pain at the enlarged lymph node sites. The most common sites for lymphadenopathy are the neck and mediastinum. Definitive diagnosis requires excisional lymph node
biopsy demonstrating Reed-Sternberg cells. The most common subtype is nodular sclerosing, and the worst prognosis occurs in the setting of the lymphocyte-depleted subtype.

Incorrect Answers: A, B, C, D, E, F, H, I, J, K, L, and M.

Acute lymphocytic leukemia (ALL) (Choice A) most commonly occurs in pediatric patients and presents with B symptoms and hepatosplenomegaly. It is defined as greater than 20% lymphoblasts on peripheral blood smears and
bone marrow biopsies. Down syndrome patients demonstrate an increased risk for the development of ALL.

Cat-scratch disease (Choice B) is caused by transmission of Bartone/la hense/ae through scratches from domestic or feral cats, typically presenting with lymphadenitis in the lymphatic drainage pattern of the scratch location.

Chronic lymphocytic leukemia (Choice C) most commonly occurs in adults greater than 60 years of age. Patients are often asymptomatic and demonstrate slow progression, with characteristic "smudge" cells on peripheral blood
smears.

Drug reaction (Choice D) would not typically present with a 1-month duration of malaise, fevers, and night sweats. While it can present with pruritus and diffuse rash, this is typically following the administration of the causative
drug.

Gastric carcinoma (Choice E) may present with early satiety and abdominal pain along with generalized malaise.

HIV infection (Choice F) can present with systemic constitutional symptoms during the acute HIV syndrome and following the clinical latent period. This patient does not demonstrate behaviors or features placing him at risk for HIV
infection.

Infectious mononucleosis (Choice H) commonly presents with fever, lymphadenopathy, and hepatosplenomegaly along with pharyngitis. Additionally, the lymphadenopathy typically involves the posterior cervical lymph nodes.

Rubella (Choice I) is a togavirus resulting in fever, arthralgias, and lymphadenopathy (most often postauricular) with a confluent rash that spreads from the head to the trunk and then centrifugally to the extremities.

Sarcoidosis (Choice J) is a chronic noncaseating granulomatous disease, most commonly presenting with bilateral hilar lymphadenopathy and possible coarse, upper lobe-predominant reticular pulmonary opacities.

Syphilis (Choice K) demonstrates multiple stages with varying symptoms, including primary with a painless chancre, secondary with fever, lymphadenopathy, and condylomata lata, and tertiary with tabes dorsalis, aortitis, and
gummas.

Systemic lupus erythematosus (Choice L) is an autoimmune disease resulting in a constellation of symptoms including fever, rash, photosensitivity, renal disease, and nonerosive arthritis.

Tuberculosis (Choice M) is a mycobacterial infection which can present with systemic symptoms including fever, night sweats, and weight loss, but is typically associated with a cough and/or hemoptysis.

r " , ~ r-- r ,
Previous Next Score Report
https://t.me/USMLENBME2CK Lab Values Calculator Help Pause
Exam Section : Item 27 of 50 National Board of Medical Examiners
■ Mark Medicine Self-Assessment

X 27. A 62-year-old man is admitted to the hospital for treatment of pneumonia. Ceftriaxone therapy is started . He also takes a thiazide diuretic for hypertension, glyburide for type 2 diabetes mellitus, and acetaminophen for
osteoarthritis. Two days after admission, laboratory studies show:
Serum
Urea nitrogen 50 mg/dl
Creatinine 2.4 mg/dl
Urine
Na+ <10 mEq/L
Fractional excretion of Na• <1
Protein trace

Urine sediment is unremarkable. Which of the following is the most likely cause of his renal failure?

A) Acute glomerulonephritis
B) Acute tubular necrosis
C) Diabetic nephropathy
D) Interstitial nephritis
E) Prerenal azotemia
F) Urethral obstruction
Correct Answer: E.

Prerenal azotemia refers to an acute kidney injury secondary to decreased renal blood flow, which results in a decreased glomerular filtration rate (GFR), typically in the setting of hypovolemia, hypotension, or renal
hypoperfusion. In response to the decreased renal blood flow, renin and angiotensin are upregulated, which stimulates increased absorption of sodium in the proximal convoluted tubule. This leads to decreased
fractional excretion of sodium (less than 1% ). The decreased GFR results in decreased tubular flow rate, which consequently leads to increased urea nitrogen reabsorption, although creatinine is unable to be
reabsorbed by the renal tubules. This results in an increased serum urea nitrogen (BUN) to serum creatinine (Cr) ratio, typically greater than 20. Treatment of prerenal azotemia requires treatment of the underlying
cause of reduced renal blood flow. If untreated, this can progress to worsening acute kidney injury and possible intrinsic renal failure (end-stage kidney disease).

lncorrectAnswers:A, B, C, D, and F.

Acute glomerulonephritis (Choice A), acute tubular necrosis (Choice B), diabetic nephropathy (Choice C) and interstitial nephritis (Choice D) are all causes of intrinsic renal failure (end-stage kidney disease), which
would present with an increased fractional excretion of sodium and a BUN/Cr ratio less than 20, due to underlying tubular dysfunction.

Acute glomerulonephritis (Choice A) refers to a variety of glomerular diseases, including nephritic and nephrotic syndromes. Nephritic syndromes typically present with acute end-stage kidney disease with
associated hematuria, red blood cell casts, and hypertension. Nephrotic syndrome typically presents with excessive proteinuria (greater than 3 g/day), hyperlipidemia, hypoalbuminemia, and edema.

Acute tubular necrosis (Choice B) typically occurs following an ischemic or nephrotoxic insult to the kidneys, which results in necrosis of the tubular epithelium. Granular, muddy brown casts are typical on urinalysis.

Diabetic nephropathy (Choice C) occurs following nonenzymatic glycosylation of the glomerular basement membrane and efferent arterioles, characteristically presenting as Kimmelstiel-Wilson lesions on light
microscopy. This progresses over time in patients with diabetes mellitus, initially beginning as microalbuminuria, which can subsequently progress to macroalbuminuria, and then end-stage kidney disease.

Interstitial nephritis (Choice D) commonly presents as a hypersensitivity reaction following exposure to certain medications such as penicillin, diuretics, and sulfonamides. It can also occur in the setting of
autoimmune diseases such as systemic lupus erythematosus. It typically presents with pyuria, predominantly eosinophilic.

• I ., e I e I I e • , I I • I I I • I .. .. I • 11 .. I II I I I I I I • I .. • I Ii I .1 I II I • I .. I I • • I • I ., .. I • 11 .. I I I • I I Ii I , I II I ., • ., • I I • I I • I I 11 .. I

r " , ~ r-- r ,
Previous Next Score Report
https://t.me/USMLENBME2CK Lab Values Calculator Help Pause
Exam Section : Item 27 of 50 National Board of Medical Examiners
■ Mark
. -. . ...... - .. ,~ . . ..... ,- . Medicine Self-Assessment

Serum
Urea nitrogen 50 mg/dl
Creatinine 2.4 mg/dl
Urine
Na+ <10 mEq/L
Fractional excretion of Na• <1
Protein trace

Urine sediment is unremarkable. Which of the following is the most likely cause of his renal failure?

A) Acute glomerulonephritis
B) Acute tubular necrosis
C) Diabetic nephropathy
D) Interstitial nephritis
E Prerenal azotemia
F) Urethral obstruction
Correct Answer: E.

Prerenal azotemia refers to an acute kidney injury secondary to decreased renal blood flow, which results in a decreased glomerular filtration rate (GFR), typically in the setting of hypovolemia, hypotension, or renal
hypoperfusion. In response to the decreased renal blood flow, renin and angiotensin are upregulated, which stimulates increased absorption of sodium in the proximal convoluted tubule. This leads to decreased
fractional excretion of sodium (less than 1% ). The decreased GFR results in decreased tubular flow rate, which consequently leads to increased urea nitrogen reabsorption, although creatinine is unable to be
reabsorbed by the renal tubules. This results in an increased serum urea nitrogen (BUN) to serum creatinine (Cr) ratio, typically greater than 20. Treatment of prerenal azotemia requires treatment of the underlying
cause of reduced renal blood flow. If untreated, this can progress to worsening acute kidney injury and possible intrinsic renal failure (end-stage kidney disease).

lncorrectAnswers:A, B, C, D, and F.

Acute glomerulonephritis (Choice A), acute tubular necrosis (Choice B), diabetic nephropathy (Choice C) and interstitial nephritis (Choice D) are all causes of intrinsic renal failure (end-stage kidney disease), which
would present with an increased fractional excretion of sodium and a BUN/Cr ratio less than 20, due to underlying tubular dysfunction.

Acute glomerulonephritis (Choice A) refers to a variety of glomerular diseases, including nephritic and nephrotic syndromes. Nephritic syndromes typically present with acute end-stage kidney disease with
associated hematuria, red blood cell casts, and hypertension. Nephrotic syndrome typically presents with excessive proteinuria (greater than 3 g/day), hyperlipidemia, hypoalbuminemia, and edema.

Acute tubular necrosis (Choice B) typically occurs following an ischemic or nephrotoxic insult to the kidneys, which results in necrosis of the tubular epithelium. Granular, muddy brown casts are typical on urinalysis.

Diabetic nephropathy (Choice C) occurs following nonenzymatic glycosylation of the glomerular basement membrane and efferent arterioles, characteristically presenting as Kimmelstiel-Wilson lesions on light
microscopy. This progresses over time in patients with diabetes mellitus, initially beginning as microalbuminuria, which can subsequently progress to macroalbuminuria, and then end-stage kidney disease.

Interstitial nephritis (Choice D) commonly presents as a hypersensitivity reaction following exposure to certain medications such as penicillin, diuretics, and sulfonamides. It can also occur in the setting of
autoimmune diseases such as systemic lupus erythematosus. It typically presents with pyuria, predominantly eosinophilic.

Urethral obstruction (Choice F) would result in postrenal azotemia from obstruction of the urinary outflow from the bladder. Postrenal azotemia occurs when outflow from bilateral kidneys is obstructed from an
underlying pathology such as bladder cancer, prostate cancer or hyperplasia, urethral stricture, or bilateral ureteral fibrosis or stricture.

r " , ~ r-- r ,
Previous Next Score Report
https://t.me/USMLENBME2CK Lab Values Calculator Help Pause
Exam Section : Item 28 of 50 National Board of Medical Examiners
■ Mark Medicine Self-Assessment

X 28. A previously healthy 42-year-old woman comes to the physician 2 days after she noted a mildly tender bump on her neck. She has not had fever, chills , cough , sore throat, or palpitations. She has smoked approximately one-half
pack of cigarettes daily for 20 years. She underwent a hysterectomy 2 years ago. Her only medication is conjugated estrogen. She is in no acute distress. She is 178 cm (5 ft 10 in) tall and weighs 57 kg (125 lb); BMI is 18 kg/m 2•
Her temperature is 37°C (98.6°F), pulse is 82/min, and blood pressure is 120/78 mm Hg . Examination of the neck shows a 2-cm lesion on the left lobe of the thyroid gland. There is no cervical lymphadenopathy. Her serum
thyroid-stimulating hormone concentration is 1.5 µU/ml, and serum free thyroxine concentration is 1.4 ng/dl (N=0.9-2.1). Which of the following is the most appropriate next step in management?

A) Reexamination and repeat thyroid function tests in 1 month


B) Measurement of thyroid radioactive iodine uptake
C) CT scan of the neck
D) Fine-needle aspiration of the lesion
E) Partial thyroidectomy
CorrectAnswer: D.

Fine-needle aspiration of the lesion is indicated for thyroid nodules greater than 1 cm in dimension with either a normal or high TSH. The patient presents with a 2-cm lesion in the left thyroid gland on physical
examination, with normal TSH and free thyroxine (FT4) levels. This indicates the nodule is not hyperfunctioning, which increases its risk for being malignant. Because of this increased risk, the appropriate next step is
a fine-needle aspiration to better characterize the underlying pathology.

Incorrect Answers: A, B, C, and E.

Reexamination and repeat thyroid function tests in 1 month (Choice A) would be indicated for subacute thyroiditis, which is a self-limiting inflammatory thyroiditis presenting with a tender thyroid and often preceded by
upper respiratory symptoms.

Measurement of thyroid radioactive iodine uptake (Choice B) is indicated for the evaluation of a thyroid nodule in the setting of low TSH, indicating a hyperfunctioning nodule. A diagnostic radioactive iodine uptake
exam helps differentiate various hyperthyroid causes such as toxic adenoma, multinodular goiter, and Graves disease.

CT scan of the neck (Choice C) would only be indicated for evaluation of a thyroid nodule if there is concern for invasion of surrounding structures (e.g., dysphagia, hoarseness), which may be seen in the setting of
some thyroid malignancies, most commonly related to anaplastic thyroid carcinoma.

Partial thyroidectomy (Choice E) is indicated for the treatment of a toxic adenoma, debulking of a large goiter resulting in compression of adjacent structures, or for differentiation of follicular nodules (e.g., follicular
adenoma versus carcinoma).

r " , ~ r-- r ,
Previous Next Score Report
https://t.me/USMLENBME2CK Lab Values Calculator Help Pause
Exam Section : Item 29 of 50 National Board of Medical Examiners
■ Mark Medicine Self-Assessment

X 29. A 47-year-old man comes to the emergency department 18 hours after the sudden onset of pain and swelling of the left knee. He has no history of recent trauma or injury to the knee. He has had two similar episodes 3 years
ago and 3 months ago; both episodes resolved spontaneously. He has a 3-year history of hypertension treated with enalapril. He has smoked one pack of cigarettes daily for 20 years . He has a 10-year history of daily alcohol
use. He is 173 cm (5 ft 8 in) tall and weighs 95 kg (210 lb); BMI is 32 kg/m 2• His temperature is 37.?°C (99.9°F), pulse is 88/min, respirations are 16/min, and blood pressure is 142/88 mm Hg . Examination shows moderate pain
and swelling of the left knee; range of motion of the knee is limited by pain. Examination of the right knee shows no abnormalities. Which of the following is the most likely diagnosis?

A) Gout
B) Osteoarthritis
C) Psoriatic arthritis
D) Rheumatoid arthritis
E) Septic arthritis
F) Systemic lupus erythematosus
Correct Answer: A.

Gout is an acute monoarticular arthropathy resulting from an intra-articular inflammatory reaction to precipitation of monosodium urate crystals within the joint space. This can result from overproduction of uric acid but
more commonly occurs following the underexcretion of uric acid. It typically presents with acute, atraumatic single-joint pain, erythema, and swelling, which can recur in patients that are under- or un-treated. The most
common joint involved is the first metatarsophalangeal, but other common joints include the knees and elbows. Treatment consists of nonsteroidal anti-inflammatory drugs (NSAIDs), colchicine, and/or corticosteroids
for acute exacerbations. Chronic preventive management involves the use of xanthine oxidase inhibitors such as allopurinol. Additionally, patients should avoid triggering risk factors such as purine-rich meals and
heavy alcohol consumption.

Incorrect Answers: B, C, D, E, and F.

Osteoarthritis (Choice B) is a noninflammatory arthropathy secondary to deterioration of articular cartilage, often related to multifactorial causes with an interplay of systemic and local factors. It typically involves large
weight-bearing joints in the body, most commonly the knees and hips.

Psoriatic arthritis (Choice C) is a subtype of seronegative spondyloarthritis associated with psoriasis. It is typically asymmetric and often presents with dactylitis and involvement of the distal interphalangeal joints.

Rheumatoid arthritis (Choice D) is an autoimmune inflammatory arthropathy that results in symmetric involvement of multiple joints, most commonly the metacarpophalangeal and proximal interphalangeal joints of the
hands, demonstrating articular erosions and progressive joint space narrowing. Ulnar deviation of the fingers and boutonniere and swan neck deformities commonly occur.

Septic arthritis (Choice E) results from bacterial infection of the synovial joint space, most commonly by Staphylococcus aureus. The affected joint demonstrates pain, swelling, and erythema. Arthrocentesis is
indicated for the evaluation of monoarticular erythema and swelling to evaluate for septic arthritis, which characteristically is purulent and demonstrates WBCs greater than 50,000/mm 3•

Systemic lupus erythematosus (Choice F) can result in a nonerosive arthropathy, commonly affecting the hands, feet, elbows, and knees. It less commonly presents as an acute monoarticular arthropathy.

r " , ~ r-- r ,
Previous Next Score Report
https://t.me/USMLENBME2CK Lab Values Calculator Help Pause
Exam Section : Item 30 of 50 National Board of Medical Examiners
■ Mark Medicine Self-Assessment

X 30. A 47-year-old woman is brought to the emergency department 24 hours after the onset of severe abdominal pain, nausea, and vomiting . She says her pain improves somewhat with vomiting and rates her current pain as a 9
on a 10-point scale. At first, her vomitus consisted of partially digested food , but now it contains yellow fluid . She also has noticed increased abdominal girth. Her last bowel movement was 24 hours ago. She has not had fever
or chills . She underwent an endodontic surgical procedure 4 days ago and has been taking acetaminophen with codeine for pain . She underwent an appendectomy at the age of 17 years and a total abdominal hysterectomy at
the age of 45 years for dysfunctional uterine bleeding. She appears uncomfortable. Her temperature is 37°C (98.6°F), and respirations are 17/min. Her pulse is 120/min and blood pressure is 110/82 mm Hg while supine; her
pulse is 130/min and blood pressure is 100/76 mm Hg while sitting . Examination shows dry oral mucosa. The abdomen is distended and tympanitic; there is mild diffuse tenderness to palpation . Bowel sounds are high pitched.
Rectal examination shows no stool in the rectal vault and no masses. Laboratory studies show:
Hematocrit 47%
Leukocyte count 13,000/mm 3
Platelet count 450,000/mm 3
Serum
Na + 145 mEq/L
K+ 3.4 mEq/L
cI - 103 mEq/L
HCO 3- 35 mEq/L
Urea nitrogen 62 mg/dL
Creatinine 1.8 mg/dL

An abdominal x-ray shows distended loops of small bowel with air-fluid levels. There is no air in the rectum. Which of the following is the most likely diagnosis?

A) Cecal volvulus
B) lieus
C Obstruction of the small bowel
D) Pseudo-obstruction of the colon
E) Sigmoid volvulus
Correct Answer: C.

Obstruction of the small bowel occurs from partial or complete blockage of the small bowel lumen and typically presents with nausea, vomiting, and abdominal pain. Partial small bowel obstructions will allow a
degree of continued flatus with diminished stooling, and complete obstructions will result in obstipation. Small bowel obstruction (SBO) has many potential causes, although the most common are post-surgical
peritoneal adhesions and hernias. On physical exam, SBO typically presents with abdominal tenderness, distention, tympany to percussion, and borborygmi (high-pitched, hyperactive bowel sounds). Potential
complications of SBO include bowel rupture and/or necrosis, which can result in sudden-onset peritonitic abdominal signs such as rebound tenderness and guarding. Abdominal x-rays typically demonstrate multiple
air-fluid levels in the setting of dilated small bowel loops. Management requires bowel rest and intravenous hydration and sometimes surgical intervention. If the patient has a partial obstruction, the patient should be
monitored and provided supportive care. If the patient demonstrates features concerning for a complete obstruction, an exploratory laparotomy should be performed to alleviate the cause of the obstruction.

lncorrectAnswers:A, B, D, and E.

Cecal volvulus (Choice A) results from twisting of the cecal mesentery and can be a cause of large bowel obstruction. It is less common than sigmoid volvulus and can be associated with a hyper-mobile cecum.

lieus (Choice B) refers to intestinal hypomotility without a mechanical obstruction, most commonly occurring following surgery or in the setting of electrolyte abnormalities, and typically presents with diffuse small
and large bowel dilatation.

Pseudo-obstruction of the colon (Choice D), also known as Ogilvie syndrome, results in dilatation of the large bowel without a mechanical obstruction. It most commonly occurs in elderly patients or patients with
severe comorbidities and/or electrolyte derangements.
- -
r " , ~ r-- r ,
Previous Next Score Report
https://t.me/USMLENBME2CK Lab Values Calculator Help Pause
Exam Section : Item 30 of 50 National Board of Medical Examiners
■ Mark Medicine Self-Assessment

or chills . She underwent an endodontic surgical procedure 4 days ago and has been taking acetaminophen with codeine for pain . She underwent an appendectomy at the age of 17 years and a total abdominal hysterectomy at
the age of 45 years for dysfunctional uterine bleeding. She appears uncomfortable. Her temperature is 37°C (98.6°F), and respirations are 17/min. Her pulse is 120/min and blood pressure is 110/82 mm Hg while supine; her
pulse is 130/min and blood pressure is 100/76 mm Hg while sitting . Examination shows dry oral mucosa. The abdomen is distended and tympanitic; there is mild diffuse tenderness to palpation . Bowel sounds are high pitched.
Rectal examination shows no stool in the rectal vault and no masses. Laboratory studies show:
Hematocrit 47%
Leukocyte count 13,000/mm3
Platelet count 450,000/mm3
Serum
Na+ 145 mEq/L
K+ 3.4 mEq/L
cI - 103 mEq/L
HCO3- 35 mEq/L
Urea nitrogen 62 mg/dL
Creatinine 1.8 mg/dL

An abdominal x-ray shows distended loops of small bowel with air-fluid levels. There is no air in the rectum . Which of the following is the most likely diagnosis?

A) Cecal volvulus
B) lieus
C Obstruction of the small bowel
D) Pseudo-obstruction of the colon
E) Sigmoid volvulus
Correct Answer: C.

Obstruction of the small bowel occurs from partial or complete blockage of the small bowel lumen and typically presents with nausea, vomiting, and abdominal pain. Partial small bowel obstructions will allow a
degree of continued flatus with diminished stooling, and complete obstructions will result in obstipation. Small bowel obstruction (SBO) has many potential causes, although the most common are post-surgical
peritoneal adhesions and hernias. On physical exam, SBO typically presents with abdominal tenderness, distention, tympany to percussion, and borborygmi (high-pitched, hyperactive bowel sounds). Potential
complications of SBO include bowel rupture and/or necrosis, which can result in sudden-onset peritonitic abdominal signs such as rebound tenderness and guarding. Abdominal x-rays typically demonstrate multiple
air-fluid levels in the setting of dilated small bowel loops. Management requires bowel rest and intravenous hydration and sometimes surgical intervention. If the patient has a partial obstruction, the patient should be
monitored and provided supportive care. If the patient demonstrates features concerning for a complete obstruction, an exploratory laparotomy should be performed to alleviate the cause of the obstruction.

lncorrectAnswers:A, B, D, and E.

Cecal volvulus (Choice A) results from twisting of the cecal mesentery and can be a cause of large bowel obstruction. It is less common than sigmoid volvulus and can be associated with a hyper-mobile cecum.

lieus (Choice B) refers to intestinal hypomotility without a mechanical obstruction, most commonly occurring following surgery or in the setting of electrolyte abnormalities, and typically presents with diffuse small
and large bowel dilatation.

Pseudo-obstruction of the colon (Choice D), also known as Ogilvie syndrome, results in dilatation of the large bowel without a mechanical obstruction. It most commonly occurs in elderly patients or patients with
severe comorbidities and/or electrolyte derangements.

Sigmoid volvulus (Choice E) results from twisting of the sigmoid mesentery, which can lead to large bowel obstruction and potential rupture or intestinal necrosis if not treated. It typically demonstrates a coffee-bean
appearance on abdominal x-rays. It is less common than a typical small bowel obstruction from post-surgical adhesions.

r " , ~ r-- r ,
Previous Next Score Report
https://t.me/USMLENBME2CK Lab Values Calculator Help Pause
Exam Section : Item 31 of 50 National Board of Medical Examiners
■ Mark Medicine Self-Assessment

31 . A 58-year-old man comes to the physician because of extreme fatigue and malaise for 3 weeks. He has felt well except for a toothache 5 weeks ago treated with a root canal procedure. He has a history of a cardiac murmur first
noted at the age of 19 years . His temperature is 37.8°C (100°F), pulse is 110/min, and blood pressure is 120/80 mm Hg . The lungs are clear to auscultation. Cardiac examination shows a grade 2/6, systolic ejection murmur
heard best at the right second intercostal space, an S"' and an ejection click. Laboratory studies show:
Hemoglobin 9.3 g/dL
Leukocyte count 10,000/mm 3
Segmented neutrophils 90%
Bands 10%
Erythrocyte sedimentation rate 90 mm/h
Urine blood positive

Blood cultures are obtained. Which of the following is the most likely underlying cardiac abnormality?

A) Calcification of a bicuspid aortic valv


B) Calcified mitral annulus
C) Myxomatous degeneration of the aortic valve
D) Myxomatous degeneration of the mitral valve
E) Myxomatous degeneration of the tricuspid valve
F) Rheumatic disease of the mitral valve
G) Rheumatic disease of the tricuspid valve
Correct Answer: A.

Calcification of a bicuspid aortic valve can lead to the development of aortic stenosis, which is characterized by a systolic crescendo-decrescendo murmur heard best at the right second intercostal space. Bicuspid
aortic valves exhibit two leaflets instead of the normal three. It is congenital in nature and can be associated with genetic disorders [e.g., gonadal dysgenesis 45,X (Turner syndrome)]. Bicuspid aortic valves result in
altered biomechanics of the valve, increasing the risk for valve degeneration at an earlier age. This valve degeneration results in dystrophic calcification over time, limiting the biomechanics of aortic valve opening, and
resulting in aortic stenosis. This degeneration of the valve increases the patient's risk for acute and/or subacute endocarditis, especially after a dental procedure. Additionally, severe aortic stenosis can result in
hemolytic anemia from red blood cell lysis as they traverse the valve, which may explain the patient's anemia and hemoglobinuria.

Incorrect Answers: B, C, D, E, F, and G.

A calcified mitral annulus (Choice B) involves dystrophic calcification of the fibrous annulus of the mitral valve. This increases the risk for development of mitral stenosis, which is evidenced by a diastolic murmur best
heard in the left fourth or fifth intercostal space along the midclavicular line.

Myxomatous degeneration of the aortic valve (Choice C), mitral valve (Choice D), and tricuspid valve (Choice E) can be primarily or secondarily related to an underlying connective tissue disorder (eg, Ehlers-Danlos
syndrome, Marfan syndrome). Myxomatous degeneration increases the risk for regurgitant murmurs of the involved valve.

Rheumatic disease of the mitral valve (Choice F) and tricuspid valve (Choice G) may be acute or chronic in nature. Acute rheumatic heart disease is a sequela from infection with group A streptococci and most
commonly affects the mitral valve. Acute rheumatic disease can lead to incompetence of the involved valves and late sequelae of chronic rheumatic disease can result in stenosis of the involved valves. Neither mitral
nor tricuspid stenosis would result in a systolic murmur in the right second intercostal space.

r " , ~ r-- r ,
Previous Next Score Report
https://t.me/USMLENBME2CK Lab Values Calculator Help Pause
Exam Section : Item 32 of 50 National Board of Medical Examiners
■ Mark Medicine Self-Assessment

X 32. A 37-year-old man comes to the physician because of a 1-year history of progressive shortness of breath. He now has shortness of breath after walking 200 feet. He coaches a soccer team and is unable to keep up with his
players on the field . He occasionally has wheezing and a nonproductive cough. He does not smoke or drink alcohol. His wife resumed smoking 18 months ago after quitting for 3 years . He has three healthy children . His father
died of alcoholic cirrhosis at the age of 55 years. The patient's temperature is 36.9°C (98.5°F), pulse is 76/min , respirations are 20/min , and blood pressure is 124/76 mm Hg. Pulse oximetry on room air shows an oxygen
saturation of 90%. End-expiratory wheezes and occasional rhonchi are heard bilaterally. The remainder of the examination shows no abnormalities. An x-ray of the chest shows increased lucency and bullous changes at both
lung bases. Which of the following is the most likely cause of this patient's condition?

A) a 1-Antitrypsin deficiency
B) Exposure to asbestos
C) Exposure to radon
D) Exposure to secondhand smoke
E) Pulmonary fibrosis
Correct Answer: A.

a 1-Antitrypsin deficiency results in the early development of panacinar emphysema and liver disease. It is a hereditary disease resulting from misfolding of a 1-antitrypsin, which aggregates in hepatocytes. This results
in hepatocyte injury and progression to cirrhosis at a young age. As well, because of the lack of functional a 1-antitrypsin release into the systemic circulation, there is an imbalance of proteases and anti-proteases in
the lung parenchyma, resulting in the destruction of alveolar septa and development of panacinar emphysema. Panacinar emphysema is characterized by emphysematous destructive changes of pulmonary alveolar
tissue beginning in bilateral lung bases resulting in bullous changes of the lungs, with the potential to spread diffusely. In patients with a 1-antitrypsin deficiency, the development of both cirrhosis and panacinar
emphysema can occur without significant additional risk factors for each disease.

Incorrect Answers: B, C, D, and E.

Exposure to asbestos (Choice B) commonly occurs in the setting of roofing, plumbing, and shipbuilding. It increases the risk for pulmonary asbestosis, characterized by the development of pleural calcified plaques
and potential lower lobe predominant interstitial fibrotic changes, not panacinar emphysema. It also increases the risk for bronchogenic carcinoma and mesothelioma.

Exposure to radon (Choice C) is the second leading cause of lung cancer following smoking. Radon occurs naturally in the environment, potentially exposing individuals within their homes, especially the basement.

Exposure to secondhand smoke (Choice D) is also a risk factor for the development of lung cancer. This patient presents with symptoms of panacinar emphysema, not lung cancer, which may present with cough,
hemoptysis, and weight loss.

Pulmonary fibrosis (Choice E) has many different causes such as idiopathic pulmonary fibrosis, sarcoidosis, and chemotherapeutics such as bleomycin, and results in a restrictive lung pathology. It typically presents
with dyspnea, dry crackles, and shallow breathing, along with symptoms related to the underlying causative pathology.

r " , ~ r-- r ,
Previous Next Score Report
https://t.me/USMLENBME2CK Lab Values Calculator Help Pause
Exam Section : Item 33 of 50 National Board of Medical Examiners
■ Mark Medicine Self-Assessment

X 33. A 42-year-old woman is brought to the emergency department because of a 1-day history of fever, chills, headache, and joint and muscle pain. She says that she was recently hospitalized in another city for treatment of an
overactive thyroid and was given two medications on discharge. She recalls no further details. Her temperature is 38°C (100.4°F), pulse is 88/min, respirations are 18/min, and blood pressure is 132/70 mm Hg. Examination
shows lid lag , exophthalmos, a prominent goiter, and a supple neck. The lungs are clear to auscultation. There are no skin or joint abnormalities. Laboratory studies show:
Hematocrit 45%
Leukocyte count 1400/mm 3
Segmented neutrophils 5%
Lymphocytes 95%
Platelet count 287,000/mm 3

Which of the following is the most likely diagnosis?

A) Autoimmune neutropenia
B) Cyclic neutropenia
C) Drug-induced neutro enia
D) Radioactive iodine-induced neutropenia
E) Thyroid storm
Correct Answer: C.

Drug-induced neutropenia is an adverse effect of both propylthiouracil and methimazole, which are thioamides used in the treatment of hyperthyroidism. The patient exhibits symptoms of hyperthyroidism secondary to
Graves disease, including exophthalmos, goiter, and lid lag. Propylthiouracil and methimazole inhibit thyroid peroxidase, while propylthiouracil also inhibits peripheral conversion of T 4 to T 3 by blocking 5'-deiodinase. A
rare adverse effect of these medications is agranulocytosis, resulting in insufficient production of segmented neutrophils. This results in susceptibility to a variety of infections and increased risk for sepsis in the setting
of neutropenic fever.

lncorrectAnswers:A, B, D, and E.

Autoimmune neutropenia (Choice A) is a rare hematologic disorder characterized by autoantibody-induced destruction of neutrophils. This patient exhibits symptoms of hyperthyroidism for which she received
medication therapy and developed the adverse effect of agranulocytosis.

Cyclic neutropenia (Choice B) is a rare disease resulting in recurrent episodes of neutropenia, typically every 3 weeks. Patients can also present with symptoms of lethargy, fever, and mucosal ulcerations.

Radioactive iodine-induced neutropenia (Choice D) can occur due to myelosuppressive effects of radioactive iodine, although this is rare given the low doses used for the treatment of hyperthyroidism secondary to
Graves disease. The patient received medical therapy for her hyperthyroidism, not radioactive iodine therapy.

Thyroid storm (Choice E) is a serious potential complication of hyperthyroidism following an acute stress such as trauma or infection. Characteristic presenting symptoms of thyroid storm include fever, diarrhea,
tachyarrhythmia, altered mental status (eg, delirium, agitation), and potential coma or death if not emergently treated. Management requires treatment with intravenous corticosteroids, beta-blocker, propylthiouracil,
and potassium iodide.

r " , ~ r-- r ,
Previous Next Score Report
https://t.me/USMLENBME2CK Lab Values Calculator Help Pause
Exam Section : Item 34 of 50 National Board of Medical Examiners
■ Mark Medicine Self-Assessment

X 34. A previously healthy 32-year-old woman comes to the physician because of a 1-week history of progressive sensory loss. Initially, only her feet felt numb, but the loss of sensation has slowly ascended symmetrically to the
umbilicus. She also has had urinary urgency and frequency, nocturia, and the perception of a tight band-like sensation around her midabdominal region . Examination shows slowing of left eye adduction during saccadic
movement of the eyes to the right. There is diffuse hyperreflexia. Sensation to pinprick is decreased to the level of the umbilicus. The most likely explanation for the slowing of left eye adduction is a lesion at which of the
following sites?

A) Left cerebral hemisphere


B) Left medial longitudinal fasciculus
C) Left paramedian pontine reticular formation
D) Right cerebellar hemisphere
E) Right sixth cranial nerve nucleus
Correct Answer: B.

The left medial longitudinal fasciculus (MLF) allows for the communication between the right abducens nerve nucleus and left oculomotor nerve nucleus. This allows for the coordinated horizontal movement in the
same direction for both eyes. The patient presents with multiple focal neurologic deficits including sensory loss, urinary symptoms, and hyperreflexia. In a younger female, neurologic deficits occurring in separate
neurologic pathways over time raises suspicion for multiple sclerosis. The most common lesion affecting the MLF is an inflammatory plaque from multiple sclerosis. A lesion affecting the left MLF would result in
impaired left eye adduction during movement of the eyes to the right, due to impaired signal to the left oculomotor nerve nucleus.

lncorrectAnswers:A, C, D, and E.

Left cerebral hemisphere (Choice A) lesions may result in a variety of neurologic deficits ranging from motor and sensory deficits involving the right side of the body and aphasia, depending on the location of the lesion
and the side of the patient's dominant hemisphere. If the lesion involves the left frontal eye fields, the patient would present with conjugate gaze deviation toward the lesion.

A lesion within the left paramedian pontine reticular formation (PPRF) (Choice C) would result in conjugate movement of both eyes away from the side of the lesion. For example, a left PPRF lesion would result in
conjugate gaze deviation to the right.

Right cerebellar hemisphere (Choice D) lesions would result in ipsilateral extremity ataxia, intention tremors, and loss of balance toward the side of the lesion.

Right sixth cranial nerve nucleus lesions (Choice E) would result in loss of motor output to the lateral rectus, resulting in a horizontal gaze palsy characterized by medial ipsilateral eye deviation.

r " , ~ r-- r ,
Previous Next Score Report
https://t.me/USMLENBME2CK Lab Values Calculator Help Pause
Exam Section : Item 35 of 50 National Board of Medical Examiners
■ Mark Medicine Self-Assessment

X 35. A 20-year-old woman comes to the emergency department because of a severe sore throat, hoarseness, and dysphagia for 8 hours. She appears toxic. Her temperature is 38.9°C (102°F). She has severe pharyngitis and
strider. Laboratory studies show leukocytosis. Which of the following is the most appropriate next step in diagnosis?

A) Assay for diphtheria toxin


B) Culture and Gram stain of sputum
C) X-ray of the chest
D) Barium swallow
E) Laryngoscopy and endotracheal intubation
Correct Answer: E.

Laryngoscopy and endotracheal intubation are the most appropriate initial steps for epiglottitis. Epiglottitis refers to acute infection of the epiglottis. It was commonly caused by Haemophilus influenzae type b prior to
widespread pediatric vaccination. Epiglottitis presents with fever and acute severe pharyngitis, drooling, hoarseness, and dysphagia. On lateral neck x-rays, it appears as a "thumbprint sign." Patients typically present
in a toxic state and can exhibit rapid respiratory decompensation with the development of strider and airway obstruction. Because of this, first-line therapy is to emergently establish a protected airway through
endotracheal intubation. In the setting of epiglottitis, endotracheal intubation should take place in the operating room under direct visualization with a laryngoscope if the patient is stable enough to permit transfer out
of the emergency department.

Incorrect Answers: A, B, C, and D.

Assay for diphtheria toxin (Choice A) would evaluate for infection by Corynebacterium diphtheriae, which can present with acute pharyngitis and production of gray oropharyngeal pseudomembranes. Complications of
diphtheria include myocarditis and neuropathy.

Culture and Gram stain of sputum (Choice B) can be used for the characterization and speciation of respiratory infections.

X-ray of the chest (Choice C) is appropriate in the evaluation of primary pulmonary pathology but offers no pertinent findings in acute pharyngitis.

Barium swallow (Choice D) evaluates pharyngeal swallowing function and timing and is utilized to evaluate causes of dysphagia. It should not be used to evaluate acute pharyngeal pathology.

r " , ~ r-- r ,
Previous Next Score Report
https://t.me/USMLENBME2CK Lab Values Calculator Help Pause
Exam Section : Item 36 of 50 National Board of Medical Examiners
■ Mark Medicine Self-Assessment

The response options for the next 2 items are the same. Select one answer for each item in the set.

For each patient with pulmonary infection, select the most likely infectious agent.

A) Cytomegalovirus
B) Haemophi/us influenzae
C) Klebsiel/a pneumoniae
D) Legionella pneumophila
E Mycop_/asma p_neumoniae
F) Pneumocystis jiroveci (formerly P. carinit)
G) Pseudomonas aeruginosa
H) Respiratory syncytial virus
I) Streptococcus pyogenes (group A)

36. A 21-year-old woman comes to the physician because of an 8-day history of sore throat, cough , and fever. The cough is nonproductive but keeps her awake at night. She subsequently develops hoarseness. Her temperature
is 38.3°C (100.9°F). Crackles are heard over the right posterior lower chest. X-rays of the chest show patchy, lower right alveolar infiltrates with increased interstitial markings. Her leukocyte count is 10,000/mm 3 with 75%
segmented neutrophils and 1% bands.
Correct Answer: E.

Mycop/asma pneumoniae is the most likely infectious agent for this patient's infection. The patient presents with a fever, nonproductive cough, and crackles along the right posterior lower chest. Her x-ray demonstrates patchy,
lower right alveolar infiltrates along with increased interstitial markings. This constellation of findings is consistent with a community acquired pneumonia secondary to an atypical infection, especially given the patient's young
age. There are a number of viruses and bacteria that can result in the development of an atypical pneumonia, such as Chlamydophi/a pneumoniae, Mycop/asma pneumoniae, Legionella pneumophila, and influenza. The most
common infectious agent for pneumonia in a patient of this age would be Mycop/asma pneumoniae, which is best treated with a macrolide antibiotic.

Incorrect Answers: A, B, C, D, F, G, H, and I.

Cytomegalovirus (Choice A), also known as human herpesvirus-5 (HHV-5) can be transmitted through multiple modes, including sexual contact, urine, respiratory droplets, and transplacental. It can cause a variety of
presentations, including mononucleosis in immunocompetent patients, and retinitis, infectious esophagitis, and pneumonia in immunocompromised patients.

Haemophilus influenzae (Choice B) is an encapsulated gram-negative bacterium that can result in various mucosal infections such as conjunctivitis, otitis media, bronchitis, and pneumonia. It was the leading cause of
epiglottitis in children prior to widespread immunization.

Klebsiella pneumoniae (Choice C) is also an encapsulated gram-negative bacterium that can present with lobar pneumonia, especially in patients with diabetes mellitus or alcohol use disorder. It can also cause nosocomial
urinary tract infections.

Legionella pneumophila (Choice D) is a gram-negative rod that is transmitted primarily through aerosols from water sources. It can lead to Legionnaires disease, characterized by fever, pneumonia, and gastrointestinal
symptoms. Legionnaires most commonly occurs in chronic smokers or those with chronic underlying lung disease.

Pneumocystis jirovecii (formerly P. carinit) (Choice F) is a yeast-like fungus that primarily affects immunosuppressed patients. It results in a bilateral pneumonia consisting of diffuse reticulogranular (ground-glass) opacities on
imaging. Treatment consists of trimethoprim-sulfamethoxazole (TMP-SMX), atovaquone, or pentamidine, with TMP-SMX serving as first-line preventive therapy for HIV patients with a CD4+ count less than 200/mm 3.

r " , ~ r-- r ,
Previous Next Score Report
https://t.me/USMLENBME2CK Lab Values Calculator Help Pause
Exam Section : Item 36 of 50 National Board of Medical Examiners
■ Mark Medicine Self-Assessment

D) Legionella pneumophila
E Mycop_/asma p_neumoniae
F) Pneumocystis jiroveci (formerly P. carinit)
G) Pseudomonas aeruginosa
H) Respiratory syncytial virus
I) Streptococcus pyogenes (group A)

36. A 21-year-old woman comes to the physician because of an 8-day history of sore throat, cough , and fever. The cough is nonproductive but keeps her awake at night. She subsequently develops hoarseness. Her temperature
is 38.3°C (100.9°F). Crackles are heard over the right posterior lower chest. X-rays of the chest show patchy, lower right alveolar infiltrates with increased interstitial markings. Her leukocyte count is 10,000/mm 3 with 75%
segmented neutrophils and 1% bands.
Correct Answer: E.

Mycop/asma pneumoniae is the most likely infectious agent for this patient's infection. The patient presents with a fever, nonproductive cough, and crackles along the right posterior lower chest. Her x-ray demonstrates patchy,
lower right alveolar infiltrates along with increased interstitial markings. This constellation of findings is consistent with a community acquired pneumonia secondary to an atypical infection, especially given the patient's young
age. There are a number of viruses and bacteria that can result in the development of an atypical pneumonia, such as Chlamydophi/a pneumoniae, Mycop/asma pneumoniae, Legionella pneumophila, and influenza. The most
common infectious agent for pneumonia in a patient of this age would be Mycop/asma pneumoniae, which is best treated with a macrolide antibiotic.

Incorrect Answers: A, B, C, D, F, G, H, and I.

Cytomegalovirus (Choice A), also known as human herpesvirus-5 (HHV-5) can be transmitted through multiple modes, including sexual contact, urine, respiratory droplets, and transplacental. It can cause a variety of
presentations, including mononucleosis in immunocompetent patients, and retinitis, infectious esophagitis, and pneumonia in immunocompromised patients.

Haemophilus influenzae (Choice B) is an encapsulated gram-negative bacterium that can result in various mucosa! infections such as conjunctivitis, otitis media, bronchitis, and pneumonia. It was the leading cause of
epiglottitis in children prior to widespread immunization.

Klebsiella pneumoniae (Choice C) is also an encapsulated gram-negative bacterium that can present with lobar pneumonia, especially in patients with diabetes mellitus or alcohol use disorder. It can also cause nosocomial
urinary tract infections.

Legionella pneumophila (Choice D) is a gram-negative rod that is transmitted primarily through aerosols from water sources. It can lead to Legionnaires disease, characterized by fever, pneumonia, and gastrointestinal
symptoms. Legionnaires most commonly occurs in chronic smokers or those with chronic underlying lung disease.

Pneumocystis jirovecii (formerly P. carinit) (Choice F) is a yeast-like fungus that primarily affects immunosuppressed patients. It results in a bilateral pneumonia consisting of diffuse reticulogranular (ground-glass) opacities on
imaging. Treatment consists of trimethoprim-sulfamethoxazole (TMP-SMX), atovaquone, or pentamidine, with TMP-SMX serving as first-line preventive therapy for HIV patients with a CD4+ count less than 200/mm 3.

Pseudomonas aeruginosa (Choice G) is a gram-negative rod that can result in numerous clinical infectious conditions, including urinary tract infections, osteomyelitis, otitis externa, folliculitis, hospital or ventilator-associated
pneumonia, and sepsis.

Respiratory syncytial virus (RSV) (Choice H) commonly causes respiratory tract infections in infants such as bronchiolitis or pneumonia. Palivizumab can be utilized for the prevention of RSV-associated pneumonia in
premature infants.

Streptococcus pyogenes (group A) (Choice I) is a gram-positive bacterium that can result in various diseases, most commonly cellulitis and pharyngitis. Additional conditions related to Streptococcus pyogenes include scarlet
fever, necrotizing fasciitis, glomerulonephritis, and rheumatic fever.

r " , ~ r-- r ,
Previous Next Score Report
https://t.me/USMLENBME2CK Lab Values Calculator Help Pause
Exam Section : Item 37 of 50 National Board of Medical Examiners
■ Mark Medicine Self-Assessment

X For each patient with pulmonary infection, select the most likely infectious agent.

A) Cytomegalovirus
B) Haemophi/us influenzae
C) Klebsiel/a pneumoniae
D Legionella pneumop_hila
E) Mycop/asma pneumoniae
F) Pneumocystis jiroveci (formerly P. carinit)
G) Pseudomonas aeruginosa
H) Respiratory syncytial virus
I) Streptococcus pyogenes (group A)

37. In September, 5 days after returning from a business trip , a 57-year-old woman develops fever, abdominal cramps , and watery diarrhea followed by a cough 24 hours later. She has smoked one pack of cigarettes daily for 20
years . Her temperature is 39°C (102.2°F), and pulse is 90/min. Crackles are heard diffusely over both lung fields. Her leukocyte count is 16,000/mm 3 with a predominance of segmented neutrophils. X-rays of the chest show
alveolar infiltrates in the right upper and lower lobes and in the left lower lobe. Gram stain of sputum shows many segmented neutrophils but no predominant microorganism.
CorrectAnswer: D.

Legionella pneumophila is a gram-negative rod that is transmitted primarily through aerosols from water sources (e.g., air conditioning systems). It can lead to Legionnaires disease, most commonly occurring in patients who
are chronic smokers or have underlying chronic lung disease. Legionnaires disease is characterized by fever, atypical pneumonia, and gastrointestinal symptoms such as diarrhea and abdominal pain. The atypical pneumonia
may appear as diffuse, patchy alveolar and interstitial opacities on lung imaging. It can also be associated with hyponatremia. Additionally, in healthy adults Legionella pneumophila can cause mild flu-like symptoms and a
fever, which is called Pontiac fever. First-line therapy for Legionella pneumophila includes administration of either macrolides or respiratory fluoroquinolones.

Incorrect Answers: A, B, C, E, F, G, H, and I.

Cytomegalovirus (Choice A), also known as human herpesvirus-5 (HHV-5) can be transmitted through multiple modes, including sexual contact, urine, respiratory droplets, and transplacental. It can cause a variety of
presentations, including mononucleosis in immunocompetent patients, and retinitis, infectious esophagitis, and pneumonia in immunocompromised patients.

Haemophilus influenzae (Choice B) is an encapsulated gram-negative bacterium that can result in various mucosal infections such as conjunctivitis, otitis media, bronchitis, and pneumonia. It was the leading cause of
epiglottitis in children prior to widespread immunization.

Klebsiella pneumoniae (Choice C) is also an encapsulated gram-negative bacterium that can present with lobar pneumonia, especially in patients with diabetes mellitus or alcohol use disorder. It can also cause nosocomial
urinary tract infections.

Mycop/asma pneumoniae (Choice E) is a common atypical bacterial cause of community acquired pneumonia in young adults, resulting in the development of alveolar and interstitial infiltrates on chest imaging.

Pneumocystis jirovecii (formerly P. carinit) (Choice F) is a yeast-like fungus that primarily affects immunosuppressed patients. It results in a bilateral pneumonia consisting of diffuse reticulogranular (ground-glass) opacities on
imaging. Treatment consists of trimethoprim-sulfamethoxazole (TMP-SMX), atovaquone, or pentamidine, with TMP-SMX serving as first-line preventive therapy for HIV patients with a CD4+ count less than 200/mm 3.

Pseudomonas aeruginosa (Choice G) is a gram-negative rod that can result in numerous clinical infectious conditions, including urinary tract infections, osteomyelitis, otitis externa, folliculitis, hospital or ventilator-associated
pneumonia, and sepsis.

r " , ~ r-- r ,
Previous Next Score Report
https://t.me/USMLENBME2CK Lab Values Calculator Help Pause
Exam Section : Item 37 of 50 National Board of Medical Examiners
■ Mark Medicine Self-Assessment

C) Klebsiel/a pneumoniae
D Legionella pneumop_hila
E) Mycop/asma pneumoniae
F) Pneumocystis jiroveci (formerly P. carinit)
G) Pseudomonas aeruginosa
H) Respiratory syncytial virus
I) Streptococcus pyogenes (group A)

37. In September, 5 days after returning from a business trip , a 57-year-old woman develops fever, abdominal cramps , and watery diarrhea followed by a cough 24 hours later. She has smoked one pack of cigarettes daily for 20
years . Her temperature is 39°C (102.2°F), and pulse is 90/min. Crackles are heard diffusely over both lung fields. Her leukocyte count is 16,000/mm 3 with a predominance of segmented neutrophils. X-rays of the chest show
alveolar infiltrates in the right upper and lower lobes and in the left lower lobe. Gram stain of sputum shows many segmented neutrophils but no predominant microorganism.
CorrectAnswer: D.

Legionella pneumophila is a gram-negative rod that is transmitted primarily through aerosols from water sources (e.g., air conditioning systems). It can lead to Legionnaires disease, most commonly occurring in patients who
are chronic smokers or have underlying chronic lung disease. Legionnaires disease is characterized by fever, atypical pneumonia, and gastrointestinal symptoms such as diarrhea and abdominal pain. The atypical pneumonia
may appear as diffuse, patchy alveolar and interstitial opacities on lung imaging. It can also be associated with hyponatremia. Additionally, in healthy adults Legionella pneumophila can cause mild flu-like symptoms and a
fever, which is called Pontiac fever. First-line therapy for Legionella pneumophila includes administration of either macrolides or respiratory fluoroquinolones.

Incorrect Answers: A, B, C, E, F, G, H, and I.

Cytomegalovirus (Choice A), also known as human herpesvirus-5 (HHV-5) can be transmitted through multiple modes, including sexual contact, urine, respiratory droplets, and transplacental. It can cause a variety of
presentations, including mononucleosis in immunocompetent patients, and retinitis, infectious esophagitis, and pneumonia in immunocompromised patients.

Haemophilus influenzae (Choice B) is an encapsulated gram-negative bacterium that can result in various mucosa! infections such as conjunctivitis, otitis media, bronchitis, and pneumonia. It was the leading cause of
epiglottitis in children prior to widespread immunization.

Klebsiella pneumoniae (Choice C) is also an encapsulated gram-negative bacterium that can present with lobar pneumonia, especially in patients with diabetes mellitus or alcohol use disorder. It can also cause nosocomial
urinary tract infections.

Mycop/asma pneumoniae (Choice E) is a common atypical bacterial cause of community acquired pneumonia in young adults, resulting in the development of alveolar and interstitial infiltrates on chest imaging.

Pneumocystis jirovecii (formerly P. carinit) (Choice F) is a yeast-like fungus that primarily affects immunosuppressed patients. It results in a bilateral pneumonia consisting of diffuse reticulogranular (ground-glass) opacities on
imaging. Treatment consists of trimethoprim-sulfamethoxazole (TMP-SMX), atovaquone, or pentamidine, with TMP-SMX serving as first-line preventive therapy for HIV patients with a CD4+ count less than 200/mm 3.

Pseudomonas aeruginosa (Choice G) is a gram-negative rod that can result in numerous clinical infectious conditions, including urinary tract infections, osteomyelitis, otitis externa, folliculitis, hospital or ventilator-associated
pneumonia, and sepsis.

Respiratory syncytial virus (RSV) (Choice H) commonly causes respiratory tract infections in infants such as bronchiolitis or pneumonia. Palivizumab can be utilized for the prevention of RSV-associated pneumonia in
premature infants.

Streptococcus pyogenes (group A) (Choice I) is a gram-positive bacterium that can result in various diseases, most commonly cellulitis and pharyngitis. Additional conditions related to Streptococcus pyogenes include scarlet
fever, necrotizing fasciitis, glomerulonephritis, and rheumatic fever.

r " , ~ r-- r ,
Previous Next Score Report
https://t.me/USMLENBME2CK Lab Values Calculator Help Pause
Exam Section : Item 38 of 50 National Board of Medical Examiners
■ Mark Medicine Self-Assessment

X 38. A 19-year-old man comes to the physician because of a 5-day history of pain with urination and a watery urethral discharge. He has not had fever. He has no history of sexually transmitted diseases. He was sexually active with
a female partner 3 weeks ago and another female partner 2 weeks ago, and he did not use condoms. He is not sure if his partners have had genitourinary symptoms. His temperature is 37 .1°C (98.8°F), pulse is 80/min, and
blood pressure is 122/70 mm Hg. Examination shows no lymphadenopathy. The penis and testes appear normal. There is a mucopurulent, whitish urethral discharge. Rectal examination shows a normal-sized , nontender
prostate. A Gram stain of the urethral discharge shows segmented neutrophils without intracellular organisms. Which of the following is the most likely causal organism?

A) Chlamydia trachomatis
B) Herpes simplex virus 1
C) HIV-1
D) Neisseria gonorrhoeae
E) Treponema pallidum
Correct Answer: A.

Chlamydia trachomatis is a common sexually transmitted disease that can cause various symptoms such as cervicitis, pelvic inflammatory disease, epididymitis, and urethritis. Urethritis secondary to infection from
Chlamydia trachomatis typically presents with painful urination and urethral discharge. Gram stain of urine typically demonstrates a sterile pyuria, characterized by the presence of white blood cells without organisms.
Diagnosis is achieved through a urine nucleic acid amplification test. Treatment requires administration of either azithromycin or doxycycline in combination with ceftriaxone for the potential of Neisseria gonorrhoeae
coinfection.

Incorrect Answers: B, C, D, and E.

Herpes simplex virus 1 (Choice B) is typically transmitted through saliva and respiratory secretions and can present with a variety of symptoms, including herpes labialis, herpetic whitlow, encephalitis, esophagitis, and
keratoconjunctivitis. While commonly a pathogen located in respiratory and oral secretions, it does have the potential for sexual contact transmission leading to genital herpes.

HIV-1 (Choice C) is the most common strain of the human immunodeficiency virus, a retrovirus that infects human T cells, resulting in immunosuppression. It is diagnosed through the combination of an ELISA
followed by Western blot assay. It can result in an acute HIV syndrome presenting with systemic inflammatory symptoms such as fever, chills, night sweats, fatigue, weight loss, and sore throat.

Neisseria gonorrhoeae (Choice D) is a gram-negative diplococcus that can present with various symptoms such as cervicitis, pelvic inflammatory disease, Fitz-Hugh-Curtis syndrome, disseminated gonorrhea, septic
arthritis, and urethritis. Urethritis secondary to infection with Neisseria gonorrhoeae typically presents with severe mucopurulent discharge, different than the watery discharge from Chlamydia trachomatis.

Treponema pallidum (Choice E) is the causative spirochete bacterium for syphilis, which presents in multiple stages with varying symptoms, including primary with a painless chancre, secondary with fever,
lymphadenopathy, and condylomata lata, and tertiary with tabes dorsalis, aortitis, and gummas.

r " , ~ r-- r ,
Previous Next Score Report
https://t.me/USMLENBME2CK Lab Values Calculator Help Pause
Exam Section : Item 39 of 50 National Board of Medical Examiners
■ Mark Medicine Self-Assessment

X 39. A 72-year-old woman is brought to the emergency department by her daughter because of a 3-day history of progressive confusion and lethargy. She underwent bilateral hip replacement 6 years ago. She takes no
medications except for aspirin as needed for osteoarthritis. She is unable to answer questions. Her temperature is 36.2°C (97.2°F), pulse is 110/min and regular, respirations are 20/min, and blood pressure is 110/70 mm Hg.
Examination shows a supple neck. The lungs are clear to auscultation. Cardiac examination shows no murmurs or gallops. The abdomen is soft with no guarding; bowel sounds are present but hypoactive. Deep tendon
reflexes are 1+ bilaterally. Babinski sign is absent. Serum studies show:
Na • 135 mEq/L
Cl· 100 mEq/L
K+ 4.2 mEq/L
HC0 3• 22 mEq/L
Urea nitrogen 52 mg/dL
Glucose 602 mg/dL
Creatinine 2.4 mg/dL

Which of the following is the most likely diagnosis?

A) Alcoholic ketoacidosis
B) Central diabetes insipidus
C) Diabetic ketoacidosis
D) Lactic acidosis
E) Myxedema
F) Nephrogenic diabetes insipidus
G Nonketotic hyperosmolar state
H) Salicylate toxicity
I) Syndrome of inappropriate secretion of ADH (vasopressin)
Correct Answer: G.

Nonketotic hyperosmolar state is a condition of severe hyperglycemia and increased serum osmolality resulting in profound dehydration, without the abnormal production of ketones. This typically occurs in patients
with type 2 diabetes mellitus after an inciting stress event, such as an acute infection. The increased osmolality results in excessive osmotic diuresis in the kidneys, which leads to progressive, marked dehydration.
Presenting symptoms include lethargy, thirst, polyuria, altered mental status, and potential coma and death if untreated. Laboratory values typically demonstrate hyperglycemia (greater than 500 mg/dL), and
increased osmolality (greater than 320 mOsm/kg) without acidosis because of the lack of ketone production. Treatment requires aggressive fluid resuscitation with isotonic intravenous fluids and insulin.

Incorrect Answers: A, B, C, D, E, F, H, and I.

Alcoholic ketoacidosis (Choice A) is a starvation state induced by excessive alcohol intake that results in increased ketone production and an anion gap metabolic acidosis, without hyperglycemia.

Central diabetes insipidus (Choice B) results from insufficient production or release of anti-diuretic hormone by the hypothalamus and pituitary. This causes inadequate absorption of free water in the renal collecting
tubules, leading to a state of dehydration and hypernatremia.

Diabetic ketoacidosis (Choice C) typically occurs in patients with type 1 diabetes mellitus following a stress event. It presents with hyperglycemia and high anion gap metabolic acidosis.

Lactic acidosis (Choice D) can occur following multiple different causes but most commonly from hypoperfusion states such as septic shock. Anaerobic metabolism results in excessive lactic acid production, which
leads to an anion gap metabolic acidosis.
r " , ~ r-- r ,
Previous Next Score Report
https://t.me/USMLENBME2CK Lab Values Calculator Help Pause
Exam Section : Item 39 of 50 National Board of Medical Examiners
■ Mark Medicine Self-Assessment
- - - - -- -- - -
Creatinine 2.4 mg/dL

Which of the following is the most likely diagnosis?

A) Alcoholic ketoacidosis
B) Central diabetes insipidus
C) Diabetic ketoacidosis
D) Lactic acidosis
E) Myxedema
F) Nephrogenic diabetes insipidus
G Nonketotic hyperosmolar state
H) Salicylate toxicity
I) Syndrome of inappropriate secretion of ADH (vasopressin)
Correct Answer: G.

Nonketotic hyperosmolar state is a condition of severe hyperglycemia and increased serum osmolality resulting in profound dehydration, without the abnormal production of ketones. This typically occurs in patients
with type 2 diabetes mellitus after an inciting stress event, such as an acute infection. The increased osmolality results in excessive osmotic diuresis in the kidneys, which leads to progressive, marked dehydration.
Presenting symptoms include lethargy, thirst, polyuria, altered mental status, and potential coma and death if untreated. Laboratory values typically demonstrate hyperglycemia (greater than 500 mg/dL), and
increased osmolality (greater than 320 mOsm/kg) without acidosis because of the lack of ketone production. Treatment requires aggressive fluid resuscitation with isotonic intravenous fluids and insulin.

Incorrect Answers: A, B, C, D, E, F, H, and I.

Alcoholic ketoacidosis (Choice A) is a starvation state induced by excessive alcohol intake that results in increased ketone production and an anion gap metabolic acidosis, without hyperglycemia.

Central diabetes insipidus (Choice B) results from insufficient production or release of anti-diuretic hormone by the hypothalamus and pituitary. This causes inadequate absorption of free water in the renal collecting
tubules, leading to a state of dehydration and hypernatremia.

Diabetic ketoacidosis (Choice C) typically occurs in patients with type 1 diabetes mellitus following a stress event. It presents with hyperglycemia and high anion gap metabolic acidosis.

Lactic acidosis (Choice D) can occur following multiple different causes but most commonly from hypoperfusion states such as septic shock. Anaerobic metabolism results in excessive lactic acid production, which
leads to an anion gap metabolic acidosis.

Myxedema (Choice E) refers to a severe hypothyroid state and presents with edema, lethargy, hypotension, and electrolyte disturbances, not with acute onset hyperglycemia.

Nephrogenic diabetes insipid us (Choice F) can be caused by genetic or acquired causes, including mutation of the V 2-receptor in the renal collecting tubules, heavy metal intake, or adverse effects of medications,
resulting in resistance to anti-diuretic hormone.

Salicylate toxicity (Choice H) results in early onset respiratory alkalosis with late onset anion gap metabolic acidosis and would not cause hyperglycemia.

Syndrome of inappropriate secretion of ADH (vasopressin) (Choice I) results from excessive ADH production most commonly in the setting of small cell lung carcinoma or intracranial pathology. This leads to the
development of a hypotonic, euvolemic hyponatremia.

r " , ~ r-- r ,
Previous Next Score Report
https://t.me/USMLENBME2CK Lab Values Calculator Help Pause
Exam Section : Item 40 of 50 National Board of Medical Examiners
■ Mark Medicine Self-Assessment

X 40. An 85-year-old woman is brought to the emergency department by paramedics 45 minutes after being found on the floor of her apartment by a home health aide. The patient says that she tripped and fell down during the night
while going to the bathroom . She was on the floor for 7 hours before being found . She says that she has not been having any health problems. There is no evidence of trauma. Vital signs are within normal limits. Physical
examination shows no abnormalities. Results of a complete blood count and measurement of serum electrolyte concentrations are within the reference ranges. Urinalysis shows:
Specific gravity 1.025
Blood 2+
Protein trace
RBC 0/hpf
WBC 0/hpf
Nitrites negative

Which of the following is the most likely diagnosis?

A) Glomerulonephritis
B) Infective endocarditis
C) Nephrolithiasis
D) Rhabdomyolysis
E) Urinary tract infection
CorrectAnswer: D.

Rhabdomyolysis refers to the abnormal breakdown of skeletal muscle tissue that can occur from multiple causes. The most common causes of rhabdomyolysis are crush/compression injuries such as prolonged
immobility, excessive exertion, alcohol abuse, medication, and toxic substances. The breakdown of skeletal muscle myocytes results in release of myoglobin and creatine kinase into the systemic circulation.
Increased creatine kinase can be utilized as a marker to evaluate for the presence and progression of rhabdomyolysis. The abnormal release of myoglobin can cause nephrotoxicity, leading to an acute kidney injury
from acute tubular necrosis. Treatment of rhabdomyolysis requires supportive care and intravenous fluid hydration, which assists in the renal clearance of myoglobin, limiting its nephrotoxicity.

Incorrect Answers: A, B, C, and E.

Glomerulonephritis (Choice A) refers to a variety of glomerular diseases, including nephritic and nephrotic syndromes. Nephritic syndromes typically present with acute end-stage kidney disease with associated
hematuria, red blood cell urine casts, and hypertension. Nephrotic syndrome typically presents with excessive proteinuria (greater than 3 g/day), hyperlipidemia, hypoalbuminemia, and edema.

Infective endocarditis (Choice B) refers to acute or subacute bacterial infection of the cardiac valvular endocardium, most commonly from infection with Staphylococcus aureus (acute) and/or viridans streptococcus
(subacute). It can present with an array of symptoms, most commonly fever, new murmur, anemia, glomerulonephritis, Roth spots, and Osler nodes.

Nephrolithiasis (Choice C) typically presents with unilateral flank pain that is colicky and sometimes radiates to the groin with associated hematuria. The common types of nephroliths are calcium oxalate or phosphate,
ammonium magnesium phosphate, uric acid, and cystine. On urinalysis, there are typically RBCs without casts.

Urinary tract infection (Choice E) can refer to either lower tract (e.g., cystitis, urethritis) or upper tract (e.g., pyelonephritis) infections. It typically presents with dysuria and increased urinary frequency for lower tract
infections with the addition of fever, nausea, and flank pain with costovertebral angle tenderness for pyelonephritis. Urinalysis typically demonstrates increased WBCs.

r " , ~ r-- r ,
Previous Next Score Report
https://t.me/USMLENBME2CK Lab Values Calculator Help Pause
Exam Section : Item 41 of 50 National Board of Medical Examiners
■ Mark Medicine Self-Assessment

X 41. A 72-year-old woman comes to the emergency department because of a 1-day history of fever, chills, and cough. She had pneumococcal pneumonia 1 year ago. Her temperature is 39°C (102.2°F). Examination shows bronchial
breath sounds at the right lung base with increased dullness and egophony. Her leukocyte count is 87,000/mm 3 (15% segmented neutrophils, 82% lymphocytes, and 3% monocytes). A Gram stain of sputum shows
gram-positive, lancet-shaped diplococci. Which of the following is most likely to confirm this patient's deficit in host defenses?

A) Assessment of segmented neutrophil function


B) Measurement of CD4+ T-lymphocyte count
C) Measurement of serum lgE concentration
D) Measurement of T-lymphocyte count
E) Quantitative immunoglobulin assay
Correct Answer: E.

Quantitative immunoglobulin assay is indicated for the evaluation of patients presenting with recurrent infections and findings indicative of a lymphoproliferative disorder. This patient's markedly increased leukocyte
count with a predominance of lymphocytes raises concern for lymphocytic leukemia. Given the patient's demographics and history of recurrent infections during the past year, the most likely diagnosis is chronic
lymphocytic leukemia {CLL). CLL typically occurs in patients older than age 60 years and is often initially asymptomatic, with the slow progressive development of symptoms such as fatigue, fevers, chills, night
sweats, and weight loss. Smudge cells are a characteristic finding on peripheral blood smear, along with marked lymphocytosis on laboratory evaluation. CLL may also result in an increased risk for recurrent
sinopulmonary bacterial infections, as the monoclonal propagation of nonfunctional B lymphocytes most commonly results in hypogammaglobulinemia. Patients presenting with a lymphoproliferative disorder and
findings concerning for immunodeficiency should undergo a quantitative immunoglobulin assay to assess for hypogammaglobulinemia.

Incorrect Answers: A, B, C, and D.

Assessment of segmented neutrophil function {Choice A) would evaluate for the presence of diseases secondary to qualitative leukocyte dysfunction such as leukocyte adhesion deficiency and chronic granulomatous
disease.

Measurement of CD4+ T-lymphocyte count {Choice B) is utilized in the evaluation of HIV to assess the state and progression of disease. CD4+ counts dictate the necessity for prophylactic treatment against various
diseases such as P. jirovecii prophylaxis for patients with a CD4+ count less than 200/mm 3•

Measurement of serum lgE concentration {Choice C) can be used to evaluate for multiple immunodeficiency syndromes. lgE is characteristically increased in autosomal dominant hyper-lgE syndrome (Job syndrome),
while it is reduced in other B-cell and combined immunodeficiencies.

Measurement of T-lymphocyte count {Choice D) is useful in the evaluation for thymic-parathyroid aplasia. Failure of the third pharyngeal pouch to appropriately develop results in thymic-parathyroid aplasia, which
leads to insufficient T cell maturation and reduced T cell levels.

r " , ~ r-- r ,
Previous Next Score Report
https://t.me/USMLENBME2CK Lab Values Calculator Help Pause
Exam Section : Item 42 of 50 National Board of Medical Examiners
■ Mark Medicine Self-Assessment

42. A 56-year-old woman has a 1-day history of epigastric pain and vomiting. She has scleral icterus and a tender epigastrium. Laboratory studies show:
Leukocyte count 14,200/mm 3
Neutrophils 78%
Serum
Bilirubin
Total 4.4 mg/dL
Direct 3.3 mg/dL
Triglycerides 210 mg/dL
Amylase 1350 Somogyi U/dL (N=60-160)
Alkaline phosphatase 320 Bodansky U/dL (N=2.0-4.5)

Ultrasonography shows dilation of intrahepatic ducts. Which of the following is the most likely cause of her condition?

A) Alcoholism
B) Cholecystitis
C) Choledocholithiasis
D) Hepatoma
E) Pancreatic carcinoma
Correct Answer: C.

Choledocholithiasis refers to the presence of a gallstone located within the common bile duct. This can result in varying degrees of obstruction, which dictates the severity of presenting symptoms. Obstructive
choledocholithiasis results in colicky right upper quadrant or epigastric pain (biliary colic), nausea, vomiting, and increased direct bilirubin and alkaline phosphatase. Direct hyperbilirubinemia can lead to scleral icterus
and jaundice. Depending on the location of the common bile duct stone, choledocholithiasis can result in acute pancreatitis, which manifests with epigastric pain and increased pancreatic enzymes (amylase, lipase). If
acute bacterial infection of the biliary tree ensues, this can lead to fever and acute cholangitis. The primary treatment for choledocholithiasis is an endoscopic retrograde cholangiopancreatography with
sphincterotomy, followed by cholecystectomy.

lncorrectAnswers:A, B, D, and E.

Alcoholism (Choice A) is a common cause of acute pancreatitis, which can result in epigastric pain and increased amylase, although would be unlikely to result in direct hyperbilirubinemia.

Cholecystitis (Choice B) results from obstruction of the cystic duct by a gallstone with resultant bacterial infection of the gallbladder, and presents with right upper quadrant pain, nausea, and vomiting. It would not
typically present with increased liver function tests or pancreatic enzymes.

Hepatoma (Choice D), also known as hepatocellular carcinoma (HCC), can present with right upper quadrant tenderness and hepatomegaly with associated hepatic enzyme (AST/ALT) and a-fetoprotein elevation.
The main risk factors for HCC are cirrhosis and chronic hepatitis B infection.

Pancreatic carcinoma (Choice E) often presents with nonspecific symptoms such as chronic abdominal pain radiating to the back, loss of appetite, nausea, vomiting, fatigue, and weight loss. If the pancreatic
carcinoma is located in the pancreatic head/neck region, it can result in painless obstructive jaundice. Pancreatic carcinoma would not typically present with acute abdominal pain or increased pancreatic enzymes.

r " , ~ r-- r ,
Previous Next Score Report
https://t.me/USMLENBME2CK Lab Values Calculator Help Pause
Exam Section : Item 43 of 50 National Board of Medical Examiners
■ Mark Medicine Self-Assessment

X 43. A 57-year-old woman with a history of alcohol abuse is brought to the emergency department by police after she was found lying in the street. On arrival, she does not respond to noxious stimuli. Her temperature is
38.1°C (100.6°F), pulse is 124/min, respirations are 20/min, and blood pressure is 94/60 mm Hg. Examination shows poor skin turgor. The oral mucosa is dry. Laboratory studies show:
Hematocrit 42%
Serum
Urea nitrogen 58 mg/dl
Creatinine 3.4 U/L
Urine
Color reddish
Specific gravity 1.020
Blood 3+
Protein 1+
RBC 0-2/hpf
WBC 0-5/hpf
Hyaline casts 1/hpf

Which of the following is the most likely diagnosis?

A) Acute glomerulonephritis
B Acute tubular necrosis
C) Allergic interstitial nephritis
D) Glomerulosclerosis
E) Pyelonephritis
Correct Answer: B.

Acute tubular necrosis is a cause of intrinsic acute end-stage kidney disease that is most commonly the result of either ischemic or nephrotoxic insults to the kidney. This patient presents after having been found
down with hemodynamic instability and signs of hypovolemia. This increases her risk for ischemic injury to the kidneys secondary to decreased renal blood flow, which can lead to tubular epithelial cell death and
sloughing into the lumen of the tubules. This typically results in an initial oliguric phase where the patient is at increased risk for uremia, hyperkalemia, and metabolic acidosis. Following this initial phase, the patient
will then enter a polyuric recovery phase characterized by excessive urine production with associated falls in creatinine and risk for hypokalemia. Urinalysis typically demonstrates granular, muddy brown casts. The
presence of blood and protein in the urine may indicate an additional underlying pathology in this patient, such as rhabdomyolysis from compression necrosis of muscle caused by prolonged pressure against the
ground while unconscious.

lncorrectAnswers:A, C, D, and E.

Acute glomerulonephritis (Choice A) refers to a variety of glomerular diseases, including nephritic and nephrotic syndromes. Nephritic syndromes typically present with acute end-stage kidney disease with
associated hematuria, red blood cell urine casts, and hypertension. Nephrotic syndrome typically presents with excessive proteinuria (greater than 3 g/day), hyperlipidemia, hypoalbuminemia, and edema.

Allergic interstitial nephritis (Choice C) most commonly presents as a hypersensitivity reaction following exposure to certain medications such as penicillin, diuretics, and sulfonamides. It can also occur in the setting
of autoimmune diseases such as systemic lupus erythematosus. It typically presents with pyuria, predominantly eosinophilic.

Glomerulosclerosis (Choice D) refers to fibrosis of the glomeruli, which can be seen in a variety of glomerular disease, including focal segmental glomerulosclerosis (FSGS) and diabetic nephropathy. FSGS is a
nephrotic syndrome most commonly caused by sickle cell disease, opioid abuse, and HIV, characterized by segmental sclerosis of the glomeruli.

Pyelonephritis (Choice E) typically presents with fevers, nausea, vomiting, and flank pain with associated costovertebral angle tenderness on physical examination. This most commonly occurs from ascending
- - - -
r " , ~ r-- r ,
Previous Next Score Report
https://t.me/USMLENBME2CK Lab Values Calculator Help Pause
Exam Section : Item 43 of 50 National Board of Medical Examiners
■ Mark Medicine Self-Assessment

38.1°C (100.6°F), pulse is 124/min, respirations are 20/min , and blood pressure is 94/60 mm Hg. Examination shows poor skin turgor. The oral mucosa is dry. Laboratory studies show:
Hematocrit 42%
Serum
Urea nitrogen 58 mg/dl
Creatinine 3.4 U/L
Urine
Color reddish
Specific gravity 1.020
Blood 3+
Protein 1+
RBC 0-2/hpf
WBC 0-5/hpf
Hyaline casts 1/hpf

Which of the following is the most likely diagnosis?

A) Acute glomerulonephritis
B Acute tubular necrosis
C) Allergic interstitial nephritis
D) Glomerulosclerosis
E) Pyelonephritis
Correct Answer: B.

Acute tubular necrosis is a cause of intrinsic acute end-stage kidney disease that is most commonly the result of either ischemic or nephrotoxic insults to the kidney. This patient presents after having been found
down with hemodynamic instability and signs of hypovolemia. This increases her risk for ischemic injury to the kidneys secondary to decreased renal blood flow, which can lead to tubular epithelial cell death and
sloughing into the lumen of the tubules. This typically results in an initial oliguric phase where the patient is at increased risk for uremia, hyperkalemia, and metabolic acidosis. Following this initial phase, the patient
will then enter a polyuric recovery phase characterized by excessive urine production with associated falls in creatinine and risk for hypokalemia. Urinalysis typically demonstrates granular, muddy brown casts. The
presence of blood and protein in the urine may indicate an additional underlying pathology in this patient, such as rhabdomyolysis from compression necrosis of muscle caused by prolonged pressure against the
ground while unconscious.

lncorrectAnswers:A, C, D, and E.

Acute glomerulonephritis (Choice A) refers to a variety of glomerular diseases, including nephritic and nephrotic syndromes. Nephritic syndromes typically present with acute end-stage kidney disease with
associated hematuria, red blood cell urine casts, and hypertension. Nephrotic syndrome typically presents with excessive proteinuria (greater than 3 g/day), hyperlipidemia, hypoalbuminemia, and edema.

Allergic interstitial nephritis (Choice C) most commonly presents as a hypersensitivity reaction following exposure to certain medications such as penicillin, diuretics, and sulfonamides. It can also occur in the setting
of autoimmune diseases such as systemic lupus erythematosus. It typically presents with pyuria, predominantly eosinophilic.

Glomerulosclerosis (Choice D) refers to fibrosis of the glomeruli, which can be seen in a variety of glomerular disease, including focal segmental glomerulosclerosis (FSGS) and diabetic nephropathy. FSGS is a
nephrotic syndrome most commonly caused by sickle cell disease, opioid abuse, and HIV, characterized by segmental sclerosis of the glomeruli.

Pyelonephritis (Choice E) typically presents with fevers, nausea, vomiting, and flank pain with associated costovertebral angle tenderness on physical examination. This most commonly occurs from ascending
urinary tract infections. Urinalysis demonstrates increased WBCs and may contain WBC casts.

r " , ~ r-- r ,
Previous Next Score Report
https://t.me/USMLENBME2CK Lab Values Calculator Help Pause
Exam Section : Item 44 of 50 National Board of Medical Examiners
■ Mark Medicine Self-Assessment

X 44. A 67-year-old man comes to the physician as a new patient to discuss his blood pressure. He brings his medical records, which show a steady increase in systolic and diastolic blood pressure over the past 5 years . He takes no
medications. He retired from his job as an insurance executive 5 years ago. He is active, exercises three times weekly, and follows a low-cholesterol diet. His blood pressure today is 160/86 mm Hg. Examination shows no other
abnormalities. Which of the following is the most critical factor in the etiology of this patient's hypertension?

A) Decreased diameter of the vascular lumen


B) Decreased vascular coml)liance
C) Increased deposition of lipids in arterial smooth muscle
D) Increased deposition of lipofuscin in cardiac myocytes
E) Increased intimal thickening of the arteries
Correct Answer: B.

Decreased vascular compliance is the most critical factor in the cause of this patient's hypertension. The patient demonstrates an increased systolic blood pressure of 160 mm Hg and pulse pressure of 74 mm Hg.
The patient's diastolic blood pressure of 86 mm Hg is at the upper limit of normal. Increased pulse pressure can occur in the setting of various pathologies, including aortic regurgitation, obstructive sleep apnea,
hyperthyroidism, and decreased aortic compliance. Vascular compliance refers to the association between changes in blood volume and pressure, with a decreased compliance resulting in abnormally increased
pressure relative to changes in volume. Atherosclerotic calcifications of the thoracic aorta result in decreased aortic compliance and are commonly associated with modifiable risk factors such as hyperlipidemia,
smoking, and diabetes mellitus, along with nonmodifiable risk factors such as male sex and increased age. This decreased aortic compliance results in hypertension primarily with isolated systolic blood pressure
elevation and is commonly seen in older persons.

lncorrectAnswers:A, C, D, and E.

Decreased diameter of the vascular lumen (Choice A) would result in increased systemic vascular resistance, most commonly associated with cholesterol atheroma buildup (atherosclerosis) resulting in increased
intimal thickening of the arteries (Choice E), especially the arterioles. Decreased vascular lumen diameter would also be seen with increased deposition of lipids in the arterial smooth muscle (Choice C). These would
result primarily in an elevation of diastolic blood pressure.

Increased deposition of lipofuscin in cardiac myocytes (Choice D) is associated with normal aging and would not directly result in progressive hypertension unless concomitant heart failure was also present.

r " , ~ r-- r ,
Previous Next Score Report
https://t.me/USMLENBME2CK Lab Values Calculator Help Pause
Exam Section : Item 45 of 50 National Board of Medical Examiners
■ Mark Medicine Self-Assessment

X 45. A 24-year-old woman comes to the physician for a routine health maintenance examination. She feels well and has no history of serious illness. She has smoked one pack of cigarettes daily for 5 years and drinks two beers
daily. She has a sedentary lifestyle. Her mother underwent bilateral knee replacement at the age of 55 years for severe osteoarthritis. The patient is 178 cm (5 ft 10 in) tall and weighs 101 kg (223 lb); BMI is 32 kg/m 2• Her pulse
is 64/min , respirations are 18/min, and blood pressure is 148/92 mm Hg. Examination shows no abnormalities. She requests advice about how to prevent osteoarthritis. Which of the following is most likely to decrease this
patient's risk for osteoarthritis?

A) Abstinence from alcohol


B) Regular aerobic exercise
C) Weight loss
D) Calcium supplementation
E) Estrogen replacement therapy at the onset of menopause
Correct Answer: C.

Weight loss is the most likely modifiable risk factor to decrease this patient's risk for osteoarthritis. Osteoarthritis is a noninflammatory arthropathy secondary to deterioration of articular cartilage, often related to
multifactorial causes with an interplay of systemic and local factors. Obesity, defined as a body mass index greater than 30 kg/m 2, is one of the most common risk factors for the development of osteoarthritis. Excess
weight results in consistent, increased daily joint stress and degradation of the articular cartilage. Further, humeral factors from adipose cells plus leptin system derangements cause alterations in metabolism of
articular cartilage. These factors result in an increased rate of degradation.

lncorrectAnswers:A, B, D, and E.

Abstinence from alcohol (Choice A) does not directly affect the risk for developing osteoarthritis. Excessive consumption of alcohol does increase the risk for osteoporosis and increases the risk for a patient to become
overweight or obese, but it does not itself increase osteoarthritis risk.

Regular aerobic exercise (Choice B) does not demonstrate supporting evidence of increased osteoarthritis risk when performed at light to moderate activity levels and with normal joints. While it can indirectly result in
decreased osteoarthritis risk through improved fitness levels and weight loss, it does not directly decrease the risk for osteoarthritis.

Calcium supplementation (Choice D) and estrogen replacement therapy at the onset of menopause (Choice E) would assist in decreasing the risk for osseous demineralization and osteoporosis development but
would not play a role in decreasing the risk for osteoarthritis.

r " , ~ r-- r ,
Previous Next Score Report
https://t.me/USMLENBME2CK Lab Values Calculator Help Pause
Exam Section : Item 46 of 50 National Board of Medical Examiners
■ Mark Medicine Self-Assessment

X 46. A 42-year-old man is brought to the emergency department because of the sudden onset of worsening headache and left-sided weakness 1 hour ago. He has no history of serious illness and has not seen a physician for
10 years. He has smoked one pack of cigarettes daily for 25 years and drinks six beers daily. He is 180 cm (5 ft 11 in) tall and weighs 97 kg (215 lb); BMI is 30 kg/m 2• His pulse is 80/min , and blood pressure is 180/120 mm Hg.
Examination shows mild-to-moderate weakness of the left lower aspect of the face. Muscle strength is decreased in the left upper and left lower extremities. Funduscopic examination shows an arteriovenous ratio of 1:3 and
regions of mild focal spasm of some arterioles. Results of a complete blood count, platelet count, and coagulation studies are within the reference ranges. His serum total cholesterol concentration is 400 mg/dl. A CT scan of the
head shows a 1.5 x 2-cm region of hyperintensity centered at the right putamen ; there is mild surrounding lucency. Which of the following is most likely to decrease this patient's risk for a recurrent stroke of this type?

A) Alcohol cessation
B) Smoking cessation
C) Weight loss
D) Atorvastatin therapy
E) Clopidogrel therapy
F) Lisinopril therapy
Correct Answer: F.

Lisinopril therapy is an ACE inhibitor antihypertension medication, which would be the most likely intervention to reduce this patient's risk for recurrent hypertension-associated intraparenchymal hemorrhagic stroke.
The patient presents with progressively worsening headache and a focal neurologic deficit, characteristic for an intraparenchymal hemorrhage. Emergent noncontrast CT of the head is indicated for evaluation, which
in this patient demonstrates a region of hyperdensity in the right putamen, consistent with an intraparenchymal hemorrhage. The greatest risk factor for an intraparenchymal hemorrhage is poorly or uncontrolled
hypertension. Hypertension most commonly results in hemorrhages within the basal ganglia, cerebellum, thalamus, and/or pons. Initiation of antihypertensive therapy to reduce the patient's blood pressure would
decrease his risk for recurrent hemorrhagic stroke.

Incorrect Answers: A, B, C, D, and E.

Alcohol cessation (Choice A), smoking cessation (Choice B), and weight loss (Choice C), while critical to improve the overall health of the patient, less directly result in reduced risk for recurrent hemorrhagic stroke as
compared to direct management of hypertension. Improved body habitus and dietary changes may contribute to improved overall health and blood pressure, although lisinopril therapy would have a greater and faster
impact on reducing the patient's blood pressure.

Atorvastatin therapy (Choice D) would assist in decreasing the patient's cholesterol, although this would primarily decrease risk for an ischemic stroke, not recurrent hemorrhagic stroke.

Clopidogrel therapy (Choice E) would be contraindicated in a patient with a history of hemorrhagic stroke as clopidogrel is an antiplatelet medication that would increase the patient's risk for hemorrhage.

r " , ~ r-- r ,
Previous Next Score Report
https://t.me/USMLENBME2CK Lab Values Calculator Help Pause
Exam Section : Item 47 of 50 National Board of Medical Examiners
■ Mark Medicine Self-Assessment

X 47. A 32-year-old nulligravid woman comes to the physician because of irregular menses for 3 years . Menses occur at 35- to 60-day intervals. Menses previously occurred at regular 28-day intervals. Her last menstrual period was
6 weeks ago. She has no history of serious illness but has been overweight most of her adult life. She is sexually active with her husband of 3 years and uses condoms consistently. She is 163 cm (5 ft 4 in) tall and weighs 77 kg
(170 lb); BMI is 29 kg/m 2• Her blood pressure is 110/60 mm Hg . Physical examination shows moderate coarse hair over the upper lip, chin, and abdomen. Abdominal and pelvic examinations show no abnormalities. Serum
studies show a thyroid-stimulating hormone concentration of 4 µU/ml, 13-hCG concentration of less than 2 mlU/mL, and prolactin concentration of 18 ng/ml. Pelvic ultrasonography shows no abnormalities. In addition to
recommending weight loss, which of the following is the most appropriate pharmacotherapy?

A) Oral contraceptive therapy


B) Oral medroxyprogesterone therapy
C) Oral prednisone therapy
D) Oral thyroid therapy
E) Depot leuprolide injection
Correct Answer: A.

Oral contraceptive therapy is the first-line therapy for polycystic ovary syndrome (PCOS) in patients who are not attempting to conceive. The patient presents with irregular menses, hirsutism, and increased body
weight, without abnormal thyroid or prolactin lab values. This is characteristic for PCOS. PCOS demonstrates excess androgens and can present with polycystic ovaries on ultrasound, oligo/anovulation, or
clinical/biochemical signs of hyperandrogenism, of which two are required for the diagnosis. The patient demonstrates a normal pelvic ultrasound, although delineation of polycystic ovaries is not required since she
exhibits signs of oligo/anovulation and hyperandrogenism. PCOS increases the risk for metabolic syndrome and type 2 diabetes mellitus. For patients who are not attempting to conceive, first-line therapy consists of
weight loss and treatment with a combination of oral contraceptives and/or metforrnin. If the patient is attempting to conceive, therapy should include weight loss, clomiphene (to improve successful ovulation), and/or
metformin.

Incorrect Answers: B, C, D, and E.

Oral medroxyprogesterone therapy (Choice B) is a type of contraceptive consisting of only progestin. Progestin administration would not improve the patient's anovulation, which requires the combination effects of
estrogen and progestin in oral contraceptives. Additionally, adverse effects of medroxyprogesterone include weight gain and irregular periods, which would not improve the patient's PCOS symptoms.

Oral prednisone therapy (Choice C) and oral thyroid therapy (Choice D) are not indicated in the treatment of PCOS. Oral prednisone is indicated in a variety of inflammatory and autoimmune disorders, although in this
patient the adverse effects of weight gain and metabolic syndrome would be deleterious. Oral thyroid therapy is not indicated given the patient's normal TSH concentration, indicating she is euthyroid.

Depot leuprolide injection (Choice E) is a GnRH analog, which serves to downregulate GnRH receptors in the pituitary and results in decreased production of luteinizing hormone and follicle-stimulating hormone.
Leuprolide is utilized for the treatment of a variety of conditions, including precocious puberty, infertility, and endometriosis.

r " , ~ r-- r ,
Previous Next Score Report
https://t.me/USMLENBME2CK Lab Values Calculator Help Pause
Exam Section : Item 48 of 50 National Board of Medical Examiners
■ Mark Medicine Self-Assessment

X 48. A 57-year-old man is brought to the emergency department 30 minutes after a 3-minute episode of unconsciousness. The patient says that the episode began with an impending sense of feeling ill while watching television . He
quickly became confused , swayed in his chair, and fell to the floor. He had no bowel or bladder incontinence during this episode. His mental status spontaneously recovered fully within 3 minutes. He has a 10-year history of
hypertension well controlled with hydrochlorothiazide. He had a myocardial infarction 4 years ago. He is 183 cm (6 ft) tall and weighs 95 kg (210 lb); BMI is 29 kg/m 2. His pulse is 86/min , respirations are 12/min, and blood
pressure is 124/86 mm Hg . Neurologic and cardiac examinations show normal findings . An ECG shows Q waves in leads 11 , 111 , and aVF. Which of the following is the most likely explanation for these findings?

A) Hypoglycemia
B) Paroxysmal ventricular tachycardia
C) Pulmonary embolus
D) Seizure
E) Transient ischemic attack
Correct Answer: B.

Paroxysmal ventricular tachycardia (VT) refers to the sudden onset of VT, defined as three or more consecutive premature ventricular contractions. This results in a wide QRS complex tachyarrhythmia. Risk factors
include underlying structural heart disease, history of coronary artery disease, or myocardial infarction. Nonsustained VT lasts for less than 30 seconds, while sustained VT persists longer than 30 seconds and
presents with varying symptoms such as palpitations and syncope. While an episode of VT may be transient, if sustained, it may require treatment with anti-arrhythmic medications or cardioversion. Class one or three
antiarrhythmics are the frequent first-line therapy for stable, sustained VT. Synchronized cardioversion may be necessary in cases of unstable, sustained VT, and defibrillation is required for pulseless VT. Syncope in
the setting of VT typically persists for the duration of the episode.

lncorrectAnswers:A, C, D, and E.

Hypoglycemia (Choice A) is a common cause of syncope, most often in patients with insulin-dependent diabetes mellitus or an underlying insulinoma. Syncope secondary to hypoglycemia requires treatment with
dextrose and/or glucagon and would not be expected to spontaneously resolve without treatment. Additionally, the patient does not exhibit risk factors for hypoglycemia, and given his cardiac risk factors,
arrhythmogenic syncope is more likely.

Pulmonary embolus (PE) (Choice C) can result in syncope in the setting of a massive PE, which would present with tachycardia and hypotension, with possible right heart strain signs on ECG (eg, "S1Q3T3" pattern).

Seizure (Choice D), such as a generalized tonic-clonic seizure, would present with sudden onset syncope with characteristic movement dependent on the type of generalized seizure. Typically, the patient would
present in a postictal state with delayed recovery of mental status.

Transient ischemic attack (Choice E) is defined as a transient episode of neurologic dysfunction caused by focal brain, spinal cord, or retinal ischemia without infarction. Transient ischemic attack resulting in syncope
does not typically present with a postictal state.

r " , ~ r-- r ,
Previous Next Score Report
https://t.me/USMLENBME2CK Lab Values Calculator Help Pause
Exam Section : Item 49 of 50 National Board of Medical Examiners
■ Mark Medicine Self-Assessment

X 49. A 62-year-old man has a 2-day history of generalized weakness and inability to walk without assistance 7 days after admission to the intensive care unit for treatment of fever, cough , and a left upper lobe consolidation .
Treatment with ceftriaxone and azithromycin initially improved his symptoms. He has not had fever for the past 2 days. He has type 2 diabetes mellitus. His current medications are ceftriaxone, heparin , omeprazole, and insulin
glargine. A peripheral catheter is in place; there is no central access. Vital signs are within normal limits. Examination of the skin shows no lesions. On pulmonary examination, rhonchi are heard in the left upper lobe. Laboratory
studies show:
Days Day?
Hemoglobin 10.8 g/dL 10 g/dL
Leukocyte count 11 ,500/mm 3 12,000/mm 3
Platelet count 330,000/mm 3 90,000/mm 3

Which of the following is the most appropriate next step in management?

A) Add epoetin alfa to the medication regimen


B) Discontinue ceftriaxone and begin physical therapy
C) Switch from ceftriaxone to piperacillin and tazobactam therapy
D) Switch from he arin to direct thrombin inhibitor therapy
E) Switch from heparin to low-molecular-weight heparin therapy
CorrectAnswer: D.

Switch from heparin to direct thrombin inhibitor therapy is indicated for the treatment of heparin-induced thrombocytopenia (HIT). HIT typically occurs between 5 and 10 days after the initiation of heparin therapy. It
results from the development of lgG antibodies against the complex of heparin and platelet factor 4. This complex leads to the activation of platelets, which can cause thrombosis, typically venous, and consumption of
platelets. This abnormal activation yields progressively decreasing platelet counts, which typically hit their lowest point at greater than 50% from their initial level. Treatment requires cessation of all heparin-related
products (eg, unfractionated heparin and low-molecular-weight heparin). The patient should be transitioned to a direct thrombin inhibitor or direct factor Xa inhibitor to ensure adequate anticoagulation without
worsening the thrombocytopenia.

Incorrect Answers: A, B, C, and E.

Add epoetin alfa to the medication regimen (Choice A) would result in increased red blood cell production from the bone marrow, not platelet production. Epoetin alfa is an analog of erythropoietin (EPO). This is
commonly utilized in the setting of chronic end-stage kidney disease for the treatment of normocytic anemia from impaired EPO production.

Discontinue ceftriaxone and begin physical therapy (Choice B) is not the best option as ceftriaxone is necessary to treat her pneumonia, and it is less likely to cause thrombocytopenia than heparin over this abrupt
time course.

Switch from ceftriaxone to piperacillin and tazobactam therapy (Choice C) would only be indicated if the patient developed a hospital or ventilator-associated pneumonia, for which broad-spectrum antibiotics are
necessary. Rather, this patient presents with community acquired pneumonia (CAP). Ceftriaxone and azithromycin are first-line therapies for CAP.

Switch from heparin to low-molecular-weight heparin therapy (Choice E) would cause further worsening of the patient's HIT, as low-molecular-weight heparin is a subtype of heparin and would further propagate the
antibody complex formation and activation of platelets. Instead, the patient requires transition to a direct thrombin inhibitor or factor Xa inhibitor for anticoagulation.

r " , ~ r-- r ,
Previous Next Score Report
https://t.me/USMLENBME2CK Lab Values Calculator Help Pause
Answer - explanation not available on NBME website.

e/USMLENBME2CK

You might also like